You are on page 1of 78

Investment planning

1 Mark Questions

1. The bid-asked spread is best described by which one of the following statements?

(a) It is the broker's commission.


(b) It is the dealer's gross income from a transaction.
(c) It is larger for illiquid securities than it is for liquid securities.
(d) All of the above are true.
(e) None of the above.

2. How do T -bills pay interest to their investors?

(a) Coupon interest.


(b) Possible price appreciation above their discounted price.
(c) T-bills pay no interest.
(d) Difference between issue price and face value

TBs are issued at a discount to the public and the RBI for enabling the Govt to meet the needs for short-
term finance.They are issued by following an auction system.

3. Which of the following are characteristics of money market securities?

(a) They are issued by the Government, municipalities and large corporations that have high-
quality ratings.
(b) All have terms to maturity that are 270 days or less.
(c) All tend to have large amounts of purchasing power risk.
(d) Both a and b.
(e) Both band c.

T Bills have a tenure of 364 days and hence option (b), (d) and (e) are incorrect. Money market
instruments are Tresury Bills, certificates of Depost, Commercial Paper and Repos. The money market is
dominated by Government, Financial Insititutions, Banks and corporate.Individual Investor scarcely
participate in the money market directly. Hence option (c) also in incorrect.

4. A basis point is which one of the following?

(a) One Rupee, Re.1


(b) One percentage point, 1 percent
(c) One paisa, Re.0.01
(d) One one-hundredth of one percentage point, 0.01 of 1 percent,

5. Government bond is best described by which one of the following statements?

(a) It has no voting privileges.


(b) It receives no cash dividends.
(c) It may be resold at any time.
(d) All of the above.

6. Equity shareholders have which of the following rights?

(a) They can legally demand information from a corporation in which they are a shareholder
and thus gain access to its books.
(b) They can vote for the common shareholders' dividend.
(c) They can vote for the preference shareholders' dividend.
(d) All of the above.

7. Preference shareholders receive priority over common stockholders with respect to which of the
following?

Investment Planning – 19Oct 07 1


(a) Dividends cannot be paid to common stockholders unless the preference stockholders receive their
stated dividend.
(b) In the event of bankruptcy and liquidation, the preference shareholders are paid before the common
shareholders.
(c) Preference shareholders get to elect the Chairman of the corporation's Board of Directors.
(d) Both a and b are true, but c is false.
(e) All of the above are true.

8. Which one of the following equations correctly defines the dividend yield (y) from a share of common
stock?

a) y = (purchase price) + (cash dividend, if any) / purchase price


b) y = (price change) + (cash dividend, if any) / purchase price
c) y = price change /purchase price
d) y = cash dividend (if any) / purchase price

9. A preference share is

a) Pays fixed dividend


b) A marketable security
c) A debt security
d) Both a and b
e) All of the above

10. Which of the following statements best describes the convertibility of preference share?

(a) Some issues of preference share may be converted into common share at the option of the investor
any time and at a conversion ratio that never changes.
(b) Some issues of preference share may be converted into common share at the option of the investor
within a limited number of years after the preferred stock is issued.
(c) Some issues of preference share may be converted into common share at the option of the investor
only after a specified number of years have elapsed since the preference share was initially issued,
(d) All of the above are true.
(e Preference share is never a convertible security.

11. Which one of the following statements best describes corporate bonds?
(a) Bond investors are creditors of the corporation.
(b) The majority of bonds make coupon interest payments once per annum.
(c) Both a and b are true.
(d) None of the above are true.

12. A debenture trust deed is best described by which one of the following statements?
(a) It is a legal contract describing the rights of specific bondholders.
(b) It describes the duties of the Trustee.
(c) Both a and b are true.
(d) None of the above are true.

13. Debenture trust deed may contain protective clauses dealing with which of the following topics?

(a) Collateral
(b) A .sinking fund
(c) All of the above
(d) None of the above

14. The quality ratings of a corporation's bond issue are primarily determined by which of the following? V

(a) The level and trend of the issuer's financial ratios


(b) The level and structure of interest rates
(c) The issuer's financial condition and the indenture contract that governs the issuing firm
Investment Planning – 19Oct 07 2
(d) All the above

15. One bond with an AA-grade rating might pay a higher yield-to-maturity than another AA-grade bond
issued at a different time by the same corporation because of which of the following reasons?

(a) Bonds with longer maturities always pay higher rates of interest than similar bonds that have
shorter maturities.
(b) The bond market is sometimes irrational and evaluates the riskiness of some bond issues
erroneously.
(c) One bond issue is a secured one whereas the other issue is unsecured.
(d) All of the above

16. Which of the following bond quality ratings applies to default-free bonds?

(a) AAA.
(b) AA.
(c) Both b and a are default-free.
(d) None of the above is default-free.

17. A security will not earn the yield-to-maturity that was promised when the security was purchased
if which of the following conditions occurs?
(a) The issuer defaults on either the interest or principal payments.
(b) The investor sells the security prior to its maturity date.
(c) Cash flows from the security paid to the investor prior to its maturity date are held in cash or
spent on consumption goods rather than reinvested.
(d) All of the above are true.
(e) None of the above are true.

18. An investor that employed a naive buy-and-hold strategy would be employing a passive investment
management strategy.

(a) True
(b) False

19. An index fund is best described by which of the following?

(a) A mutual fund constructed to achieve a particular investment goal


(b) A portfolio that attempts to match the performance of some stock market index by investing in the
same stocks and in the same proportions as those that comprise the selected market index
(c) Both of the above
(d) An investment portfolio that appreciates in value at least as rapidly as some inflation index (such
as the Consumer Price Index, for instance)
(e) Both a and d

20. The weights used in constructing a value-weighted stock market index are best described by which of
the following statements?

(a) Equal weights are assigned to every security in the index.


(b) The weight assigned to each stock is proportionate to its price per share.
(c) The share price of every stock in the index is multiplied by the number of shares outstanding
to determine the weight of that issue based on its total value stated as a proportion of the
aggregate market value of all the stocks in the index.
(d) The weight assigned to each stock in the index is proportional to the number of shares that issue has
outstanding stated as a proportion of the aggregate number of shares outstanding for all issues that
comprise the index.
(e) Both a and d are true.

21. All stock market indexes are most accurately characterised by which of the following statements
about the degree to which they covary together?

(a) They are perfectly positively correlated.


(b) They are highly positively correlated.
Investment Planning – 19Oct 07 3
(c) They are uncorrelated.
(d) They are negatively correlated.
(e) It is impossible to generalize, some are highly positively correlated and some are negatively
correlated.

22. Depreciation must be entirely omitted from a firm's net profit in order to determine how much cash
flow the firm generated.

(a) True
(b) False

23. When a firm pays creditors the transaction does not affect the equity capital shown in its balance
sheet in any way.

a) True
b) False

24. A primary issue of bonds or stock would increase both sides of the issuing company‟s balance sheet
by the same amount.
a) True
b) False

25. The retention rate equals 100 percent less the percent of the corporation's earnings paid out for cash
dividends.

a) True
b) False

26. If a corporation has preference share outstanding its book value per share equals its total net worth
divided by the number of shares of common stock plus preference shares it has outstanding.

a) True
b) False

27. Which of the following ratios will increase as a firm uses more financial leverage?

(a) The times-interest-earned ratio


(b) The debt-to-equity ratio
(c) The inventory turnover
(d) Both a and b
(e) Both a and c

28. Which of the following factors tends to increase the growth rate of a corporation?

(a) External borrowing


(b) Increasing the retention rate
(c) Increasing the rate of return on equity
(d) Both a and b
(e) All of the above

29. A company has total assets of 2,000,000. It has 700,000 in long-term debt. If stockholders' equity is
900,000, what is its total debt to total asset ratio?

(a) 45 percent
(b) 47 percent
(c) 59 percent
(d) 52 percent
(e) 55 percent

30. A corporation had a total debt to total asset ratio of .4, total debt of Rs.200,000, and net income of
Rs.30,000. Determine the corporation's return on equity.

Investment Planning – 19Oct 07 4


(a) 8 percent
(b) 9 percent
(c) 10 percent
(d) 12 percent
(e) 14 percent

31. Assume the following information: stockholders' equity = Rs.2,000; shares outstanding = 40; market
price to book value = 2. Determine the market price for the firm's common stock.

(a) Rs.75
(b) Rs.100
(c) Rs.110
(d) Rs.115
(e) Rs.117

32. Which of the following is a source of funds?

(a) An increase in inventory


(b) An increase in accounts receivable
(c) An increase in investments
(d) An increase in accounts payable
(e) None of the above

33. Assume the correlation coefficient r between the rates of return from these two cement sector shares,
say, A and G was +0.9. If you took a long position in G and a short position in A (or vice versa) of exactly
equal value you would be perfectly hedged.

(a) True
(b) False

34. Imperfect hedges occur when either the quantities sold short and bought long are out of balance, or
the purchase and short sale prices differ.

(a) True
(b) False

35. What characteristics do the long and short positions have in common?

(a) The potential profits from a long position and the potential losses from a short position are both infinite
if the price of the underlying security rises to infinity.
(b) There is a one-to-one correspondence between movements in the price of the underlying security and
the investor's profits
(c) Both b and a are true.
(d) The investor loses money if the price of an asset held in either position declines.
(e) Both a and d are true

36. Which of the following statements correctly describes a speculator‟s (short seller) profits?

(a) The per share profit is limited to an amount equal to the price at which the shares were sold short.
(b) The short seller earns Re.1 profit for every Re.1 fall in price of the security.
(c) Short selling can help arbitragers earn profits.
(d) All the above are true.

37. Selling securities short is useful in which of the following activities?

(a) Speculating
(b) Hedging
Investment Planning – 19Oct 07 5
(c) Arbitrage
(d) All the above

38. Investors‟ motives are not correctly described by which of the following statements?

(a) Having long positions indicates that the buyer is bullish.


(b) Uncovered short sellers expect the market to be bearish.
(c) Hedgers are always bullish.
(d) Both a and b are false
(e) All of the above are false

39. Why do share prices usually drop when news about decline in a company‟s earnings per share is
reported?

(a) Because a reduction in a earnings means that the firm has less money with which to pay dividends
and therefore the market fears a reduction in the company's future dividends.
(b) Because the share market anticipates that a decreased level of earning power might be the indicator
of default and perhaps even bankruptcy.
(c) The statement is false. Share prices do not usually react to announcements about current earnings.
(d) Both a and b are true.

40. Interest-rate risk is defined by which of the following statements?

(a) Fluctuations in the coupon interest rates from one bond issue to the next
(b) Fluctuations in the market prices of bonds as their prices move inversely to the prevailing market
interest rates
(c) The variability of returns as a result of fluctuations in market interest rates
(d) Both a and b
(e) All of the above

41. Assume that you are an investment adviser and one of your clients, on your advice, invested
Rs.100,000 in Treasury bonds due to mature in 2 years. If your client becomes worried that a general
increase in the level of interest rates will reduce the market value of his bond portfolio, what should you
say to allay your client's fears?

(a) You could assuage your client's fears by claiming you foresee only stable interest rates ahead.
(b) You could instruct your client to liquidate their portfolio of Treasury bonds and reinvest the proceeds in
a bank.
(c) Both a and b are true.
(d) You could tell your client not to worry because the market prices of short-term bonds do not
fluctuate very much.

42. Assume you are an financial advisor and one of your clients reads something about interest-rate risk
and is worried that if market interest rates declined her coupon interest income will likewise decline. His
bond investments have maturities ranging from 15 to 30 years. What advice is appropriate for this client?

(a) Tell the investor to liquidate her coupon-paying bonds and reinvest the money in zero coupon bonds.
(b) Tell your client not to worry, her coupon income will not vary until her coupon bonds mature in
15 to 30 years.
(c) Both a and b are true.
(d) The client need not worry if market interest rates are expected to rise because coupon rates vary
inversely with market interest rates and therefore her coupon interest could increase.
(e) All the above are true.

43. Calculate the (1) expected rate of return, E(r), from the probability distribution of returns below for the
ABC common share.

FIVE RATES OF
POSSIBILITlES RETURN PROBABILITY

i=5 -0.5 = 50% 0.1


i=4 -0.1 = -10% 0.25
Investment Planning – 19Oct 07 6
i=3 0.2 = 20% 0.3
i=2 0.5 = 50% 0.25
i=1 0.9 = 90% 0.1
Total 1.0
The expected rate of return for ABC is which one of the following?

(a) The E(r) is 5 percent.


(b) The E(r) is 20 percent.
(c) The E(r) is 5 percent.
(d) The E(r) is 10 percent.
(e) The E(r) is 12 percent.

44. Which one of the following statements describes the phrase „risk‟?

(a) The phrase total risk is synonymous with the variability of return from an asset.
(b) Bond quality ratings essentially measure the probability that an issue of bonds falls into default.
(c) Although Treasury bonds are free from default risk they nevertheless contain substantial amounts of
interest-rate risk.
(d) Both a and b are true.
(e) All the above are true.

45. If ABS‟s price is Rs.40 per share and its current dividend of Rs.3.85 per share, which is growing at a 7
percent rate per year, determine its required return?

(a) 16.2 percent


(b) 15.1 percent
(c) 16.6 percent
(d) 17.3 percent
(e) 18.2 percent

46. If ABS‟s pays dividend of Rs.3.85 per share which is growing at a 7 percent rate per year and is
expected to grow at the same rate in future. Its required rate of return is 14.5% Determine its share value.

(a) 52.48
(b) 49.25
(c) 54.93
(d) 55.75
(e) 47.26

47. A company has current earnings per share of Rs.6. Assume a dividend-payout ratio of 55 percent.
Earnings grow at a rate of 8.5 percent per year. If the required rate of return is 15 percent, what is its
current value?

(a) Rs.51.33
(b) Rs.55.08
(c) Rs.57.02
(d) Rs.52.05
(e) Rs.50.75

48. The value of an option tends to increase as the volatility (or risks) of the underlying asset increases.

a) True
b) False

49. If you purchase a put option, you are expecting the value of the underlying asset to increase.

a) True
b) False

Directions for Questions 50 to 58: Use the information given in the table below to answer the questions.
Ignore taxes and transaction costs.

Time to Maturity
Investment Planning – 19Oct 07 7
Share Current Exercise Call Premium Put Premium
price price 3 months 6 months 3 months 6 months
A 52 50 3 4 0.35 1.05
B 40 45 1 1.25 5.5 6.00
C 35 30 6 6.3 0.45 0.65

Each contract is equal to 100 shares. .

50. If you purchase one 3-month call contract on A, what profit or loss will you make at the maturity date if
the price of A at that time is Rs.57?

(a) Rs.200
(b) Rs.400
(c) -Rs.460
(d) Rs.500
(e) Rs.560

Exercise Price X = 50. Premium Paid= Rs.3 Total Cost = Rs.53 *100 = Rs. 5300. This is the amount for
which you have the option of buying the shares today.
Market Price at the time of expiry of option = Rs.57 * 100 = Rs. 5700
Thus 5700 – 5300 = Rs. 400 is the profit for you.

51. If B's price is Rs.35 at the maturity of the 6-month option, determine the value of five 6-month put
contracts at their maturity date.

(a) Rs.2,000
(b) Rs.5,700
(c) Rs.8,200
(d) -Rs.4,000
(e) Rs.3,600

52. If you had purchased five 3-month call options of C and the price of C's share is Rs.32 at maturity.
Determine your profit or loss on the investment.

(a) Rs.1,000
(b) Rs.1,500
(c) -Rs.2,000
(d) -Rs.4,000
(e) -Rs.500

A 3 month call option for C will come at (X)Rs.30+ (Premium)Rs.6 = Rs.36. for five lots of 100 the price at
which you have the right to buy the shares is Rs. 36*100*5 = Rs.18000
Now the price of the share at maturity is Rs.32*100*5 = Rs. Rs.16000
Thus Rs.18000- Rs.16000 = Rs. 2000 is the loss on the investment.

53. If you had purchased five 3-month puts on C, what would your profit "or loss position have been at
maturity if the share's price was Rs.32?

(a) -Rs.225
(b) -Rs.400
(c) -Rs.600
(d) Rs.400
(e) Rs.600
Here the exercise price of the put option is (30* 100) – (.45*100) = Rs. 2955
Now is the current market price is Rs. 32*100 = Rs.3200 then you will incur a loss of Rs. 3200-2955 =
Rs. 245 on this transaction.

Investment Planning – 19Oct 07 8


54. Your client wrote five 6-month call options on B's share. What is his profit or loss on the options at
maturity if the price of B at that time is Rs.43?

(a) Rs.625
(b) -Rs.600
(c) Rs.400
(d) Rs.300
(a) '(e) Rs.200

55. If your client had written five 6-month put options on B, what would his profit or loss have been at the
maturity of the options if the share price was Rs.43 per share?

(a) Rs.1,000
(b) Rs.2,000
(c) Rs.1,800
(d) Rs.1,500
(e) -Rs.500

56. Which of the following options are in the money?

(a) A's 3-month call


(b) B's 6-month put
(c) C's 6-month put
(d) a and b
(e) None of the above

57. If an investor is bearish on a share, buying a put is usually better than selling short because

(a) The holder's losses can be no more that the put premium if the share price rises, but the short seller's
losses could be unlimited in this situation.
(b) The short sale will become worthless after a short period of time but the put will not become worthless.
(c) The short seller must pay any dividends paid by the security the short seller borrowed.
(d) a and b.
(e) a and c.

58. Call option-premiums for a given asset tend to increase when

(a) The price of the underlying asset decreases.


(b) The volatility of the underlying asset decreases.
(c) The time to maturity of the option increases.
(d) a andb.
(e) None of the above.

Directions for Questions 59 to 63: Choose the correct option

59. The Black Scholes model cannot be used to determine the overall market value of a firm.

(a) True
(b) False

60. When a firm's dividend payment is included in the Black Scholes model, the value of a put decreases.

(a) True
(b) False

61. Dividend payment on a firm's common share tends to lower the value of a call option on the firm's
equity.

Investment Planning – 19Oct 07 9


(a) True
(b) False

62. Option prices that are calculated with the Black Scholes model are not very sensitive to changes in
the asset's standard deviation of returns.

a) True
b) False

63. The option values calculated with the binomial model will approach those calculated with the Black
Scholes model for a given period of time as we divide the fixed time period into smaller and smaller units.

a) True
b) False

Directions for Questions 64 to 73: Using the information given below, answer the questions with the
Black Scholes option pricing model.

The options on the share of National Corporation have the following values:

S = Rs.55 k = Rs.52 R = .10 u = .33 and T = .4

Assume that no dividends are currently being paid.

64. What is the value of a call on the National Corporation?

(a) Rs.6.50
(b) Rs.6.80
(c) Rs.7.17
(d) Rs.8.05
(e) Rs.8.35

65. If the call is undervalued, what approach should an investor follow?

(a) Buy 1,000 calls, sell short 712 shares of National share.
(b) Buy 1,000 calls, sell short 600 shares of National share.
(c) Buy 500 shares of National, sell 1,000 calls.
(d) Buy 714 shares of National, sell 1,000 calls.
(e) Buy 600 shares of National, sell 1,000 calls

66. Determine the value of a put on National's share.

(a) Rs.1.75
(b) Rs.2.13
(c) Rs.2.65
(d) Rs.2.95
(e) Rs.3.15

67. If the put on National's share is overpriced, what should an investor do?

(a) Sell 1,000 puts for every 400 shares sold.


(b) Buy 1,000 puts for every 300 shares purchased.
(c) Sell 1,000 puts for every 288 shares sold.
(d) Sell 1,000 puts for every 350 shares purchased.
(e) Buy 1,000 puts for every 400 shares sold.

68.Assume the National Corporation is paying dividends at the rate of 4 percent per year. Determine the
price of a call.

(a) Rs.5.57
(b) Rs.5.85
(c) Rs.6.03
Investment Planning – 19Oct 07 10
(d) Rs.6.35
(e) Rs.6.74
.
69. Determine the price of a put on National's share with a 4 percent annual dividend.

(a) Rs.2.56
(b) Rs.2.85
(c) Rs.1.95
(d) Rs.2.40
(e) Rs.2.96

70. If the price of a National call is Rs.6.41, determine the implied standard deviation of returns without
dividends.

(a) .37
(b) .40
(c) .25
(d) .35
(e) .18

71. If R rises to .12, determine the price of a call on National's share if the other inputs do not change.
Assume no dividends.

(a) Rs.6.80
(b) Rs.7.25
(c) Rs.7.61
(d) Rs.7.90
(e) Rs.8.10

72. If T = .5, determine the value of a put on National's share if the other inputs do not change. Assume
no dividends.

(a) Rs.1.50
(b) Rs.1.75
(c) Rs.1.91
(d) Rs.2.51
(e) Rs.2.30

73. If k = Rs.54, determine the value of a call on National's share if the other inputs do not change.
Assume no dividends.

(a) Rs.5.86
(b) Rs.6.19
(c) Rs.6.30
(d) Rs.6.42
(e) Rs.6.76

74. With two assets, as the correlation coefficient between the two assets is reduced, the portfolio's risk is
reduced.

a) True
b) False

75. A 50-asset portfolio has ----- unique covariance terms.

(a) 2499
(b) 1449
(c) 1225
(d) none of the above

76 In the two-asset case, the portfolio risk-return possibilities are nonlinear when the correlation between
Investment Planning – 19Oct 07 11
the asset returns is less than + 1.

a) True
b) False

Directions for questions 77 and 78

The ex post returns of 2 shares are:


YEAR RETURN A Return B
-
1991 20% 30%
1992 -10 -20
1993 15 18
1994 17 10
1995 19 5

77. Calculate the covariance of returns.

(a) 220.32
(b) -420.11
(c) 145.22
(d) 270.36
(e) 162.08

78. Calculate the correlation coefficient from the above information.

(a) .540
(b) .869
(c) .923
(d) .758
(e) .697

Directions for questions 79 and 80: Refer to the data below

Two companies, M and N have the following risk and return statistics:
Standard deviation (M) = 18%;
Standard deviation (N) = 30%
Expected Return (M) = 14%; Expected Return (N) = 19%
Correlation coefficient = 0.28

79. Determine the risk of a portfolio of 25 percent M and 75 percent N.

(a) 18.25 percent


(b) 30.15 percent
(c) 24.15 percent
(d) 21.75 percent
(e) 27.13 percent

80. Determine risk of a portfolio of 50 % M and 50 % N from the information given in problem 81.

(a) 19.5 percent


(b) 21.7 percent
(c) 17.8 percent
(d) 23.0 percent
(e) 25.4 percent

Ans: a

81. Two assets, 3 and 4, have the following risk and return inputs:
E(r3) = 14% E(r4) = 22%
Standard deviation (3) = 19% standard deviation (4) = 32%
Correlation coefficient = -1

Investment Planning – 19Oct 07 12


Directions for questions 82 to 84: Use the information in the table below to answer the questions.
Assume the risk less rate is the average of the five annual T-bill returns.

ABC FUND'S XYZ FUND'S S&P500's T-BILL'S


TIME RETURNS % RETURNS % RETURNS % RETURNS %

19XO 15 10 11 6
19X1 -6 -2 -5 5
19X2 17 13 12 7
19X3 18 9 11 6
19X4 22 11 13 7

82. Determine the Sharpe performance index for the ABC Fund for the 5-year period.

(a) .71
(b) .68
(c) .91
(d) 1.05
(e) 1.10

83. Determine the Sharpe performance index for the XYZ Fund for the 5-year period.

(a) .48
(b) .38
(c) .78
(d) .92
(e) 1.05

84. Determine the Jensen performance measure (alpha) for the ABC Fund over the 5-year period.

(a) 3.72
(b) 2.69
(c) 1.76
(d) 2.01
(e) 3.76

85. Duration for a zero coupon bond is less than its term to maturity.

a) True
b) False

86. Longer-term bonds are almost always more volatile in terms of price than short-term bonds for a given
change in interest rates.

a) True
b) False

87. Bond price volatility is directly related to the bond's coupon.

a) True
b) False

88. Duration for a coupon-paying bond is always less than its term to maturity.

a) True
b) False

89. For any given maturity, bond price movements that result from an equal absolute decrease or
increase in the yield-to-maturity are symmetrical.

Investment Planning – 19Oct 07 13


a) True
b) False

90. There is a direct relationship between a bond's coupon and duration.

a) True
b) False

91. As a bond's YTM increases, if other things are held constant, its duration decreases.

a) True
b) False

92. When a bond is selling at a discount, its YTM exceeds the coupon rate.

a) True
b) False

93. When a bond's YTM equals its coupon rate, the bond's price is less than par value.

a) True
b) False

94. 10 percent semiannual bond with a YTM of 12 percent and 10 years to maturity has a price equal to

(a) Rs.1,051.65
(b) Rs.1,159.88
(c) Rs.885.30
(d) Rs.888.89
(e) Rs.955.41

95. The price of the bond in above Problem after 2 years, assuming everything else stays the same, is
(Hint:There will be 8 years until maturity.)

(a) Rs.1,130.55
(b) Rs.935
(c) Rs.757
(d) Rs.868
(e) Rs.898.94

96. A bond's duration measures which one of the following?

(a) The time structure of a bond's cash flows


(b) The bond's interest-rate risk
(c) Both a and b above
(d) The default risk of the bond issue
(e) None of the above

97. If the market rate of interest falls, a coupon-paying bond will

(a) Decrease in value


(b) Experience a decrease in duration
(c) Experience an increase in duration
(d) None of the above
(e) Both a and b above

98. A bond's reinvestment rate risk:

(a) Refers to the problem of being able to purchase another bond with the same or higher YTM when the
existing bond matures or is called
(b) Is the risk of not being able to reinvest the coupons of a bond at the bond's YTM
(c) Is the same as marketability risk
(d) Both a and b
Investment Planning – 19Oct 07 14
(e) None of the above

99.14 If you expect a large decline in interest rates, which of the following investments should you
choose?

(a) Money market fund


(b) Low-coupon short-term bond
(c) High-coupon short-term bond
(d) Long-term zero coupon bond
(e) Short-term zero coupon bond

100. Bonds with higher coupons, other things being the same,

(a) Have more interest-rate risk than bonds with smaller coupons
(b) Have less interest-rate risk than bonds with smaller coupons
(c) Have higher duration than smaller-coupon bonds
(d) Have lower duration than smaller-coupon bonds
(e) Both b and d

Source: Kaplan Investment Planning

Investments - Options (moderate)


101. A client with a large, well-diversified common stock portfolio expresses concern about a possible
market decline. However, he/she does not want to incur the cost of selling a portion of their holdings
nor the risk of mistiming the market. A possible strategy for him/her would be

A. Buy an index call option.


B. Sell an index call option.
C. Buy an index put option.
D. Sell an index put option.
E. He cannot protect against the decline with these options.

Solution: The client will benefit if the market increases. Her portfolio, because it is diversified, should be
highly correlated and move with the market. An index option also moves with the market and, therefore,
would be a good hedge vehicle. A put should be used, because it will increase in value if the market
should decline, thereby offsetting any losses on the portfolio.

102. The Performance Fund had returns of 19% over the evaluation period and the benchmark portfolio
yielded a return of 17% over the same period. Over the evaluation period, the standard deviation of
returns from the Fund was 23% and the standard deviation of returns from the benchmark portfolio
was 21%. Assuming a risk-free rate of return of 8%, which one of the following is the calculation of
the Sharpe index of performance for the fund over the evaluation period?

A. .3913.
B. .4286.
C. .4783.
D. .5238.
E. .5870.

Solution: Realized Return - Risk Free Return Sharp Index = Standard Deviation of Portfolio (0.19 –
0.08). 0.47826 = 0.23. Answer is C
Investment Planning – 19Oct 07 15
Investments - Standard Deviation of Portfolio (moderate)
103. The standard deviation of the returns of a portfolio of securities will be ____________ the
weighted average of the standard deviation of returns of the individual component securities.

A. Equal to.
B. Less than.
C. Greater than.
D. Less than or equal to (depending upon the correlation between securities).
E. Less than, equal to, or greater than (depending upon the correlation between
securities).

Solution: Answer is option D, unless the correlation coefficient bet ween the stocks is equal to one, the
standard deviation for the portfolio will be lower than the weighted average standard deviation for the
portfolio.

Investments - Stock Valuation (moderate)


104.According to fundamental analysis, which phrase best defines the intrinsic value of a share of
common stock?

A. The par value of the common stock.


B. The book value of the common stock.
C. The liquidating value of the firm on a per share basis.
D. The stock's current price in an inefficient market.
E. The discounted value of all future dividends.

Solution: Answer is option E; the intrinsic value of a share of common stock is equal to the discounted
present value of its cash flows.

Investments - Municipal Bonds (easy)


105. Municipal bonds that are backed by the income from specific projects are known as:

A. Income bonds.
B. Revenue bonds.
C. General obligation bonds.
D. Debenture bonds.
E. Project bonds.

Solution: Answer is option B. General obligation bonds are backed by the full taxing authority of the
municipality, whereas revenue bonds are only repaid from revenues of a particular project.

106. A call option with a strike price of 110 is selling for 3½ when the market price of the underlying
stock is 108. The intrinsic value of the call is:

A. 0.
B. 1½.
C. 2.
D. 3½.
E. (2).

Solution: The option is out of the money; therefore the intrinsic value is zero. The premium of an option
does not affect the intrinsic value of the option, hence (a)

Investments – Arbitrage Pricing Theory (easy)


107. In contrast to the Capital Asset Pricing Model, the Arbitrage Pricing Theory (APT):

A. Is usually a multi-factor model.


B. Is primarily using by arbitrageurs to profit from imperfections in security markets.
C. Assumes a market portfolio.
D. Is a useful technical indicator.

Investment Planning – 19Oct 07 16


Solution: Arbitrage Pricing Theory uses multiple regression (many factors) to determine a model or
formula that has numerous factors. This model is then used to determine the value of a security. The
CAPM is based on the single factor of Beta, which measures the level of systematic risk within a portfolio.

Investments - Options (moderate)


108. With the same Rupee investment, which of the following strategies can cause the investor
to experience the greatest loss?

A. Selling a naked put option.


B. Selling a naked call option.
C. Writing a covered call.
D. Buying a call option.
E. Buying the underlying security.

Solution: A call option has unlimited price potential which means that writing a call without the stock as a
hedge will provide the greatest loss potential.

Investments - Stock Valuation (difficult)


109. If the market risk premium were to increase, the value of common stock (everything else being
equal) would:

A. Not change because this does not affect stock values.


B. Increase in order to compensate the investor for increased risk.
C. Increase due to higher risk-free rates.
D. Decrease in order to compensate the investor for increased risk
E. Decrease due to lower risk-free rates.

Solution: The market risk premium is the additional return for accepting the risk of the market. If the
market premium increases with all else remaining the same, then the price of the stock would have to
decrease. An increase in the market premium would also increase the discount rate used to value the
stock. This higher discount rate will cause the present value of the cash flows to be smaller.

Investments - Stock Valuation (difficult)


110.The current annual dividend of ABC Corporation is Rs.2.00 per share. Five years ago the dividend
was Rs.1.36 per share. The firm expects dividends to grow in the future at the same compound
annual rate as they grew during the past five years. The required rate of return on the firm's common
stock is 12%. The expected return on the market portfolio is 14%. What is the value of a share of
common stock of ABC Corporation using the constant dividend growth model? (Round to the nearest
rupee)

A. Rs.11.
B. Rs.17.
C. Rs.25.
D. Rs.36.
E. Rs.54.

Solution: PV = Rs.1.36
FV = (Rs.2.00)
N= 5
i = 8.02
d1 1.082 (2.00) 2.16
Value of common stock = == = Rs.54.00
RRR - g 0.12 – 0.08 0.04
RRR = Required Rate of Return
g = growth rate

Investments – Performance Measures (easy)


111. In computing portfolio performance, the Sharpe index uses , while the Treynor index uses for the
risk measure.

1 Standard deviation.
2 Variance.
3 Correlation coefficient.
Investment Planning – 19Oct 07 17
4 Coefficient of variation.
5 Beta.

A. 5; 1.
B. 1; 3.
C. 1; 4.
D. 1; 5.
E. 2; 5.

Solution: Answer is option D. Sharpe uses standard deviation and Treynor uses beta.

Investments – Unit Investment Trusts (easy)

112. Which of the following is/are characteristics of a municipal bond unit investment trust?
1 Additional securities are not added to the trust.
2 Shares may be sold at a premium or discount to net asset value.
3 Shares are normally traded on the open market (exchanges).
4 The portfolio is self-liquidating.

A. 1 only.
B. 1 and 4 only.
C. 2 and 3 only.
D. 2 and 4 only.
E. 1, 2, 3, and 4.

Solution: Once capitalized, unit trusts do not accept additional funds. Unit trusts are self-liquidating.
Units are generally traded directly with the fund at net asset value.

Investments – Bond Valuation (moderate)

113. A Rs.1,000 bond originally issued at par maturing in exactly 10 years bears a coupon rate of 8%
compounded annually and a market price of Rs.,147.20. The indenture agreement provides that the
bond may be called after five years at Rs.1,050. Which of the following statements is/are true?
1 The yield to maturity is 6%.
2 The yield to call is 5.45%.
3 The bond is currently selling at a premium, indicating that market interest rates have fallen since
the issue date.
4 The yield to maturity is less than the yield to call.

A. 1, 2, and 3 only.
B. 1 and 3 only.
C. 2 and 3 only.
D. 4 only.
E. 1, 3, and 4 only.

Solution: Answer is option A


YTM YTC
PV = (Rs.1,147.20) (Rs.1,147.20)PMT= 80 80N=10 5FV = Rs.1,000 Rs.1,050i = 6 5.45

The price of a bond is inversely related to changes in interest rates. Therefore, #1, #2, and #3 are all
correct.

Investments/ Retirement/Insurance – Regulatory Requirements (easy)


114. Which of the following statements about industry/regulatory relationships are true?
1 The insurance industry is primarily regulated by each of the 50 states.
2 The majority of banks are subject to federal regulation by the Federal Reserve System and the
Federal Deposit Insurance Corporation.
3 Pension plan funds are primarily subject to federal regulation.
4 The organized stock exchanges, such as the New York Stock Exchange, are primarily regulated
by the individual states in which they are incorporated.
Investment Planning – 19Oct 07 18
A. 1, 2, and 3 only.
B. 1 and 3 only.
C. 2 and 4 only.
D. 2 and 3 only.
E. 1, 2, 3, and 4.

Solution: Answer is option A. Organized exchanges are regulated by the SEC, which is a Federal
agency. All other statements are true.

Investments – Risk (easy)


115. Which of the following are non-diversifiable risks?
1 Business risk.
2 Management risk.
3 Company or industry risk.
4 Market risk.
5 Interest rate risk.
6 Purchasing power risk.

A. 4, 5, and 6 only.
B. 1, 2, and 3 only.
C. 5, 6, and 2 only.
D. 1, 3, and 4 only.
E. 4, and 6 only.

Solution: Answer is option A. Non-diversifiable risks or systematic risks are those that affect the entire
market, including market risk, interest rate risk, and purchasing power risk.

Investments – American Depository Receipts (easy)


116. American depository receipts (ADRs) are used to:
1 Finance foreign exports.
2 Eliminate currency risk.
3 Sell U.S. securities in overseas markets.
4 Trade foreign securities in U.S. markets.

A. 1 and 3 only.
B. 1 and 4 only.
C. 2 and 4 only.
D. 4 only.
E. 1, 2, and 4 only.

Solution: Answer is option D.ADRs are used to trade foreign securities in the U.S. ADRs are foreign
shares denominated in U.S. dollars and do not eliminate currency risk.

Investments – Bond Swaps (easy)


117. Which combination of the following statements about bond swaps is true?

1 A substitution swap is designed to take advantage of a perceived yield differential between bonds
that are similar with respect to coupons, ratings, maturities, and industry.
2 Rate anticipation swaps are based on forecasts of general interest rate changes.
3 The yield pickup swap is designed to change the cash flow of the portfolio by exchanging similar
bonds that have different coupon rates.
4 The tax swap is made in order to substitute capital gains for current yield.

A. 1, 2, and 3 only.
B. 1 and 3 only.
C. 2 and 4 only.
D. 4 only.
E. 1, 2, 3, and 4.

Investment Planning – 19Oct 07 19


Solution: Answer is option A. Statements #1, #2 and #3 are true. Statement #4 is false because tax
swaps generally take advantage of capital losses by selling bonds, which have been devalued by
increasing interest rates.

Investments – Modern Portfolio Theory (moderate)


118.Modern "asset allocation" is based upon the model developed by Harry Markowitz. Which of the
following statements is/are correctly identified with this model?
1 The risk, return and covariance of assets are important input variables in
creating portfolios.
2 Negatively correlated assets are necessary to reduce the risk of portfolios.
3 In creating a portfolio, diversifying across asset types (e.g., stocks and
bonds) is less effective than diversifying within an asset type.
4 The efficient frontier is relatively insensitive to the input variable.

A. 1 and 2 only.
B. 1, 2, and 3 only.
C. 1 only.
D. 2 and 4 only.
E. 1, 2, and 4 only.

Solution: Answer is option C. Statement #1 is true. It is not necessary to have negatively correlated
assets; it is only necessary to have assets that have a correlation less than positive one (+1); thus,
statement #2 is false. Statement #3 is false, because diversifying across asset types is more, not less,
effective than within an asset type. Statement #4 is false, because all the input variables in statement #1
help to create the efficient frontier.

Investments - Municipal Bond Insurance (easy)


119. Municipal bonds are frequently insured. One of the insurers is the:
a. Federal Insurance Guarantee Corporation.
b. Resolution Trust Corporation.
c. Associated Municipal Bond Corporation.
d. Municipal Insurance Group.
e. Municipal Bond Insurance Association.

Solution: Municipal Bond Insurance Association (MBIA) is the only choice that insures municipal bonds.

Investments – Security Types (easy)


120. Which one of the following products is designed to provide both growth and income?
A. Fixed premium annuity.
B. Non-participating mortgage real estate investment trust (REIT).
C. Aggressive growth mutual fund.
D. Convertible bond.

Solution: Convertible bonds generate current income from coupon payments and allow for growth
through the stock conversion feature. Options “A” & “B” provide income only and Option “C” is designed
for growth.

Investments - Options (easy)


121.Jennifer is optimistic about the long-term growth of her Widget stock. However, the stock, currently
priced at Rs.58, has made a sharp advance in the last week and she wants to lock in a minimum
price in case the shares drop. What might Jennifer do?

A. Buy Rs.55 call options.


B. Sell Rs.55 call options.
C. Buy Rs.55 put options.
D. Sell Rs.55 put options.

Solution: Answer is option C. The exercise price for a put is the price at which you can sell the stock.
Thus, a price of Rs.55 will be assured if she buys a Rs.55 put.
Investment Planning – 19Oct 07 20
Investments – Stock Valuation (moderate)
122.Company ABC is currently trading at Rs.35 and pays a dividend of Rs.2.30. Analysts project a
dividend growth rate of 4%. Your client Tom requires a rate of 9% to meet his stated goal. Tom
wants to know if he should purchase stock in Company ABC.

A. Yes, the stock is undervalued.


B. No, the stock is overvalued.
C. No, the required rate is higher than the projected growth rate.
D. Yes, the required rate is higher than the expected rate.
E. No, the required rate is lower than the expected rate.
d0 (1 + g) Rs.2.30 (1.04)
Solution: V = = = Rs.47.84 (value using dividend growth model), k - g 0.09 - 0.04
Since the value of Rs.47.84 is greater than the current market price, the stock is underpriced in the
market. Tom should purchase the stock.

Investments – Security Types (easy)


123. Your client will only invest in securities backed by the full faith and credit of the US
government. Which of the following would you consider for his portfolio?

A. Federal Home Loan Mortgage Association Debentures (“Freddie Macs”).


B. Federal National Mortgage Association Debentures (“Fannie Maes”).
C. Government National Mortgage Association Certificates (“Ginnie Maes”).
D. Student Loan Marketing Association Notes (“Sallie Maes”).
E. Tennessee Valley Authority (TVA) Securities.

Solution: Answer is option C. Ginnie Maes are the only securities listed that are backed by the full faith
and credit of the U.S. Government.

Investments – Investment Planning Process (moderate)


124.If the client needs to accumulate wealth but is risk-averse, which of the following is the most crucial
action the planner needs to take to have the client achieve the goal of wealth accumulation?
Advise investing the client‟s current assets

A. In the products which will bring the highest return to the client regardless of risk.
B. In products that produce high income for the client because fixed income products are
generally safe.
C. In diversified mutual funds because of the protection which diversity provides.
D. After determining the client’s risk tolerance.
F. In 100% cash equivalents in the portfolio because most software programs
recommend this safe approach.

Solution: Answer is option D. One of the first steps that must be taken in planning for wealth
accumulation is to determine the risk tolerance of the investor.

125. Your client is contemplating the exchange of two parcels of investment land for two similar parcels.
Given the following details of the proposed transactions, compute the amount of recognized gain and
loss (if any) on both parcels if your client does the exchanges.

Parcel A: ten acres of land acquired 15 years ago with a current basis of Rs.50,000. In
exchange
your client will receive eight acres of land (FMV Rs.80,000) and Rs.20,000 of cash.
Parcel B: twenty acres of land acquired two years ago with a current basis of Rs.100,000. In
exchange your client will receive twelve acres of land (FMV Rs.75,000) and Rs.10,000 of cash.

Parcel A Parcel B
Recognized Gain Recognized Loss
A. Rs.20,000 Rs.0
B. Rs.20,000 Rs.10,000
C. Rs.50,000 Rs.10,000
D. Rs.20,000 Rs.15,000
E. Rs.50,000 Rs.15,000
Investment Planning – 19Oct 07 21
Solution: Answer is option A

A: New Parcel Rs.80,000Cash Received Rs.20,000Adjusted Basis of Old Parcel (50,000) Gain
Realized Rs.50,000
The client must recognize gain up to the boot received (cash) of Rs.20,000.

B :New Parcel Rs.75,000Cash Received Rs.10,000Adjusted Basis at Old Parcel (100,000)Loss


Realized (Rs.15,000)
The client will not recognize loss in a like-kind exchange. The client will have a new asset worth
Rs.75,000 with a basis of Rs.90,000 (Rs.100,000 - 10,000).

Investments – Security Valuation (moderate)


126. Which of the following would result in the largest increase in the price of a diversified common
stock mutual fund?

A. Unexpected inflation.
B. Expected dividend increases.
C. Unexpected corporate earnings growth.
D. Expected increase in the prime interest rate.

Solution: Answer is option C. Of all the choices, only unexpected corporate earning growth would have a
significant positive impact on the value of a stock. Under the Efficient Market Hypothesis, any expected
changes would already be reflected in the price of the common stock. Unexpected increases in inflation
would increase the discount rate and reduce the value of the fund.

Investments - Capital Market Line (moderate)


127. In analyzing the position of a portfolio in terms of risk/return on the capital market line (CML),
superior performance exists if the fund's position is the CML, inferior performance exists if the fund's
position is the CML, and equilibrium position exists if it is the CML.

A. Above; on; below.


B. Above; below; on.
C. Below; on; above.
D. Below; above; on.
E. On; above; below.

Solution: Answer is option B. Above the line would indicate a higher than expected return for the given
risk level. On the line would indicate an expected return for the given risk level. Below the line would
indicate lower than expected return for the given risk level.

Investments – Fixed-Income Securities (easy)


128. Which one of the following best describes a debenture?
A. A long-term corporate promissory note.
B. An investment in the debt of another corporate party.
C. A long-term corporate debt obligation with a claim against securities rather than
against physical assets.
D. A corporate debt obligation that allows the holder to repurchase the security at
specified dates before maturity.
E. Unsecured corporate debt.

Solution: Answer is option E. A debenture is an unsecured corporate debt.

Investments - Immunization (difficult)


129. A client has a cash need at the end of seven years. Which of the following investments might initially
immunize the portfolio?
1 A 9-year maturity coupon bond.
2 A 7-year maturity coupon Treasury note.
Investment Planning – 19Oct 07 22
3 A series of Treasury bills.

A. 1, 2, and 3.
B. 1 only.
C. 2 and 3.
D. 2 only.
D. 1 and 2.

Solution: Answer is option B. The goal of immunization is to match the investment time horizon with the
duration of the portfolio. Since the duration of a coupon bond is less than its maturity, only the 9-year
bond might immunize the portfolio. The other two choices will not immunize the portfolio. The best choice
would be a 7-year zero-coupon bond; however, this choice is not available.

Investments - Risk (moderate)


130. Which combination of the following statements about investment risk is correct?
1 Beta is a measure of systematic, non-diversifiable risk.
2 Rational investors will form portfolios and eliminate systematic risk.
3 Rational investors will form portfolios and eliminate unsystematic risk.
4 Systematic risk is the relevant risk for a well-diversified portfolio.
5 Beta captures all the risk inherent in an individual security.

A. 1, 2, and 5.
B. 1, 3, and 4.
C. 2 and 5.
D. 2, 3, and 4.
E. 2 and 5.

Solution: Answer is option B. Systematic risk cannot be eliminated, thus statement #2 is false. Beta only
measures systematic risk; statement #5 is false. All other statements are true.

Investments - Rupee Cost Averaging (moderate)


131. Which combination of the following statements is true regarding the investment strategy known
as “Rupee-cost averaging”?
1 Invests the same Rupee amount each month over a period of time.
2 Purchases the same number of shares each month over a period of time.
3 Lowers average cost per share over a period of time (assuming share price fluctuations).
4 Invests the same Rupee amount each month to protect the investment from loss of capital.

A. 1 and 2.
B. 1 and 3.
C. 2 and 3.
D. 2 and 4.
E. 1, 2, 3, and 4.

Solution: Answer is option B. Rupee cost averaging is when an equal rupee amount is invested
periodically. This does not prevent capital losses but can lower the average cost per share due to periods
of decline in the stock price.

Investments - Bonds (Difficult)


132. Harry Ingram purchased ten listed bonds (Widget Corp 8.00s10/1/99) on June 24, 1995, at a
market asked price of 95. His transaction cost from the trade was Rs.100. He paid his broker Rs.9,800
as a consequence of the trade. He asks you to explain the details of his purchase. You reply that:

1 His purchase cost included Rs.9,500 for the bonds, Rs.100 for broker commission and Rs.200 as
mark-up by the trade specialist.
2 His broker will report Rs.400 on Form 1099-INT as 1995 taxable interest on these bonds. His
taxable interest income from the bonds for 1995 was Rs.400.
3 His purchase cost included Rs.9,500 for the bonds, Rs.100 for broker commission and Rs.200 as
accrued interest.
4 His broker reported Rs.400 on form 1099-INT as 1995 taxable interest on these bonds. His
taxable interest income from the bonds for 1995 was Rs.200.
5 His purchase cost included Rs.9,500 for the bonds, Rs.100 for broker commission and Rs.200
Investment Planning – 19Oct 07 23
charged in error by the brokerage house.

A. 3 and 4.
B. 1 and 2.
C. 1 and 4.
D. 2 and 3.
E. 2 and 5.

Solution: Answer is option A


Rs.800 per year (annual coupon).
Rs.400 per coupon payment (semi-annual coupon).
Rs.200 represents accrued interest, because the bond was purchased at a date in the middle of the
coupon payment period (not on an interest date).
Therefore, the Rs.9,800 is comprised of the following:
Rs.9,500 Bonds100 Commission200 Accrued interest
Rs.9,800 Since he received a coupon payment of Rs.400, he will receive a Form 1099 for the same
amount. However, only Rs.200 of the Rs.400 will be taxable since only Rs.200 has been earned. Rs.200
was accrued and prepaid.

Investments - Taxation of Securities (moderate)


133. Your client is designing an educational investment program for her eight year-old son. She expects
to need the funds in about ten years when her AGI will be approximately Rs.45,000. She wants to
invest at least part of the funds in tax-exempt securities. Identify which investment(s) would yield tax-
exempt interest on her federal return if the proceeds were used to finance her son‟s education.
1 Treasury bills.
2 EE bonds.
3 GNMA funds.
4 Zero coupon Treasury bonds.

A. 3 and 4.
B. 1, 3, and 4.
C. 2 and 3.
D. 2 and 4.
E. 2 only.

Solution: Answer is option E. Of the choices, only EE bonds can generate Federal tax -exempt income.
T-Bills and T-Bonds generate state tax-exempt income but not Federal tax-exempt income. GNMA funds
are taxable at the state and Federal level.

Directions for questions 134 and 135: Refer to the caselet below and answer the questions
Smith invests in a limited partnership that requires an outlay of Rs.9,200 today. At the end of years 1
through 5, he will receive the after-tax cash flows shown below. The partnership will be liquidated at the
end of the fifth year. Smith is in the 28% tax bracket.

YEARS CASH FLOWS


0 (Rs.9,200) CFo
1 Rs.600 CF1
2 Rs.2,300 CF2
3 Rs.2,200 CF3
4 Rs.6,800 CF4
Investment Planning – 19Oct 07 24
5 Rs.9,500 CF5

Fundamentals - IRR (moderate)


134. The after-tax IRR of this investment is:
A. 17.41%.
B. 19.20%.
C. 24.18%.
D. 28.00%.
E. 33.58%.

Solution: Answer is option C


CFo = (Rs.9,200) CF1
= Rs.600 CF2 =
Rs.2,300 CF3 =
Rs.2,200 CF4 =
Rs.6,800 CF5 =
Rs.9,500 IRR =
24.18%

Fundamentals - IRR (moderate)


135. Which of the following statements is/are correct?
1 The IRR is the discount rate, which equates the present value of an investment's expected costs
to the present value of the expected cash inflows.
2 The IRR is 24.18% and the present value of the investment's expected cash flows is Rs.9,200.
3 The IRR is 24.18%. For Smith to actually realize this rate of return, the investment's cash flows
will have to be reinvested at the IRR.
4 If the cost of capital for this investment is 9%, the investment should be rejected because its net
present value will be negative.

A. 2 and 4.
B. 2 and 3.
C. 1 only.
D. 1, 2, and 3.
E. 1 and 4.

Solution: Answer is option D. Statement #4 is false, because if the cost of capital is less than the IRR,
then the project should be accepted (NPV > 0).

Directions for questions 136: Refer to the caselet below and answer the questions
The tax bracket and holdings of your client are as follows:

• Federal tax bracket = 33%


Annual June 30, Last Year June 30, This Year
Investment* Income Purchase Price Market Price
Money fund Rs.6,500 Rs.100,000 Rs.100,000
11% T bonds Rs.11,000 Rs.100,000 Rs.140,000
S&P Index fund Rs.6,000 Rs.100,000 Rs.160,000

Investment Planning – 19Oct 07 25


Computer stock fund Rs.3,000 Rs.100,000 Rs. 85,000

*There have been no capital gains distributions.


th th
136. During the 12 months from June 30 , last year, through June 30 , this year, the portfolio earned, in
annual yield and before-tax appreciation, respectively:
A. 5.5% and 17.5%.
B. 5.5% and 21.3%.
C. 6.6% and 17.5%.
D. 6.6% and 21.3%.

Solution: Answer is option D


Total annual income Rs.26,500
= = 6.63% annual yield
FMV @ June 30 last year Rs.400,000

Change in value Rs.85,000

Investments – Duration (moderate)


137.Assuming that the duration of Bond A is 1.94 years, which of the following statements about the
effect of a 1% decline in interest rates is true?

A. Bond C, having a longer duration than Bond A, would have a larger percent
increase in price than Bond A.
B. The percent change in price of a bond is independent of the duration of a bond.
C. It is not possible to determine the percent change in price of Bond A versus Bond C
because the duration of Bond C is not given.
D. Bond A would have a greater percent change in price than Bond C because it has a
shorter duration.
E. The percent change in the price of Bonds A and C is equal since it is not affected by
duration.

Solution: Answer is option A. The bond with the longest duration has the greatest interest rate risk. The
greater the duration, the more effect a change in interest rates has on the bond‟s value. A zero coupon
bond‟s duration equals its time to maturity.
FMV @ June 30 last year = Rs.400,000 = 21.25% appreciation (Before tax)

Investments – Bonds (easy – application)


138. The following set of newly issued debt instruments was purchased for a portfolio: Treasury bond
Zero-coupon bond Corporate bond Municipal bond.

The respective maturities of these investments are approximately equivalent. Which one of the
investments in the preceding set would be subject to the greatest relative amount of price volatility if
interest rates were to change quickly?

A. Treasury bond.
B. Zero-coupon bond.
C. Corporate bond.
D. Municipal bond.

Investment Planning – 19Oct 07 26


Solution: Answer is option B. A zero-coupon bond‟s duration equals its maturity. Coupon bonds (the
remaining options) have durations less than their maturities. Higher duration bonds are subject to more
price volatility than lower duration bonds.

Investments – Bonds (easy – application)


139. Which of the following best describes the investment characteristics of a high-quality long-term
municipal bond?

A. High inflation risk; low default risk.


B. Low inflation risk; high market risk.
C. Low inflation risk; low default risk.
D. High inflation risk; high market risk.

Solution: Answer is option A. A longer-term bond will be subject to more inflation risk. Since the quality of
the bond is high, the level of default risk is low.

Investments –Options (moderate)


140. Jasmine has a large paper profit in her Amalgamated Corporation shares, currently at 46. She is
happy with the stock but realizes that a good thing cannot go on forever. If she is willing to sell at 50, what
strategy could you recommend to her?

A. Buy Rs.50 call options.


B. Sell Rs.50 call options.
C. Buy Rs.50 put options.
D. Sell Rs.50 put options.

Solution: Answer is option B. Selling a call option will allow her to generate income from the option
premium with little risk since she does not expect the stock to continue to increase. If the stock does
exceed Rs.50, she would be paid what she wants for the stock.

Investments –Security Pricing Theories (easy)


141. “Stock prices adjust rapidly to the release of all new public information.” This statement is an
expression of which one of the following ideas?
A. Random walk hypothesis.
B. Arbitrage pricing theory.
C. Semi-strong form of the efficient market hypothesis.
D. Technical analysis.

Solution: Answer is option C. The statement in the question is referring to the definition of the semi
strong form of the efficient market hypothesis.

Investments – Bond Valuation (moderate)


142. Assuming that the current market yield for similar risk bonds is 8%, determine the discounted
present value of a Rs.1,000 bond with a 7.5% coupon rate, which pays interest semiannually and
matures in 17.5 years.

A. Rs.504.68.
B. Rs.539.78.
C. Rs.953.34.
D. Rs.968.96.
E. Rs.1,653.26.

Solution: Answer is option C


N = 35 (17.5 x 2)i = 4 (8  2)PMTOA = Rs.37.50 (Rs.75  2)
FV = Rs.1,000PV = (Rs.953.34)

Investments – Performance Measurements (moderate)


143. Given the following diversified mutual fund performance data, which fund had the best risk-
adjusted performance if the risk-free rate of return is 5.7%?
Fund Average Annual Standard Standard Beta
Return Deviation of Deviation of
Investment Planning – 19Oct 07 27
Annual Return Annual Return
A .0782 .0760 0.950
B .1287 .1575 1.250
C .1034 .1874 0.857
D .0750 .0810 0.300

A. Fund B because the annual return is highest.


B. Fund A because the standard deviation is lowest.
C. Fund C because the Sharpe ratio is lowest.
D. Fund D because the Treynor ratio is highest.
E. Fund A because the Treynor ratio is lowest.

Solution: Answer is option D. Generally, investors evaluate performance of investments based on risk
adjusted returns. Therefore, A and B must be wrong since they only address one aspect of the risk-return
relationship. Treynor and Sharpe ratios are performance measures in which the higher the ratio, the
better the risk adjusted return. No calculation is needed for this question. D is the only reasonable
answer. It is the only one that indicates that a fund should be chosen, because the performance measure
is high (not low).

Investments – Immunization (moderate)


144. To immunize a bond portfolio over a specific investment horizon, an investor would do which of
the following?

A. Match the maturity of each bond to the investment horizon.


B. Match the duration of each bond to the investment horizon.
C. Match the average weighted maturity of the portfolio to the investment horizon.
D. Match the average weighted duration of the bond portfolio to the investment horizon.

Solution: Answer is option D. To immunize a bond portfolio, an investor should match the duration of the
bond portfolio to the investment time horizon.

Investments – Mortgage Backed Securities (easy)


145. Mortgage-backed securities may contain which of the following risks?
1 Purchasing power risk.
2 Interest rate risk.
3 Prepayment risk.

A. 2 only.
B. 1 and 2 only.
C. 1 and 3 only.
D. 1, 2, and 3.

Solution: Answer is option D. Mortgage–backed securities are subject to the same risk
as bonds plus the risk of prepayment.

Investments – Investment Planning Process (moderate)


146.You receive a phone call from an individual you have not spoken with previously. The caller is
excited, just having heard that a new mutual fund is positioned to deliver large gains in the coming
year. The caller wishes to purchase shares of the fund through you. Keeping in mind stages of the
overall personal financial planning process, which of the following questions that addresses the first
two stages of the financial planning process should you ask the caller?
1 What are your goals for this investment?
2 What other investments do you have?
3 What is your date of birth?
4 Do you want your dividends reinvested?

A. 1 and 3 only.
B. 2 and 4 only.
C. 1, 2, and 3 only.
D. 1, 2, and 4 only.

Solution: Answer is option C. Question #1 should always be asked. Question #2 is a reasonable

Investment Planning – 19Oct 07 28


question, since it provides the planner with a concept of the client‟s investment experience leading to
suitability. This type of analysis is a part of data gathering. Question #3 goes to suitability and should be
asked, or the age should be determined relative to risk and time horizon. The question relates to the first
two stages of data collection and goals. Reinvestment of dividends does not go to suitability and will be
addressed after the investment decision is made.
Below is the CFP Board of Examiners‟ clarification of the question:
(1), (2), and (3) all relate to the second step of the financial planning process, namely, “Gathering client
data and determining goals and expectations”. The first step is “Establishing the client-planner
relationship”.

Fundamentals – Regulatory Requirements (easy)


147. Which of the following are exemptions under the definition of “investment adviser”?
1 Banks that are not investment companies.
2 Accountants or lawyers whose investment advice is “solely incidental” to the practice of their
profession.
3 Persons whose advice relates only to securities issued or guaranteed by the U.S. government.
4 Publishers of financial publications that have regular and general circulation.

A. 1 and 3 only.
B. 2 and 4 only.
C. 1, 2, and 4 only.
D. 1, 2, 3, and 4.

Solution: Answer is option D. All are exempt under the definition of “investment advisor”.

Investments – Duration (easy)


148. The duration of a bond is a function of its:
1 Current price.
2 Time to maturity.
3 Yield to maturity.
4 Coupon rate.

A. 1 and 3 only.
B. 2 and 3 only.
C. 2 and 4 only.
D. 1, 2, and 3 only.
E. 1, 2, 3, and 4.

Solution: Answer is option E. Duration is a present value, time weighted measure of payback. All of the
factors listed are important components in determining duration.

Investments – Bonds (easy)


149. You are faced with the following alternative fixed income investments.
A. A U.S. Treasury bond with an 11.625% coupon, due in 2004 with a price of
Rs.142.50 and a yield to maturity of 6.3%.
B. A U.S. Treasury strip bond (zero coupon) due in 2004 with a price of Rs.46.75 and a
yield to maturity of 6.25%.
C. A corporate B-rated bond with a 9.75% coupon, due in 2004 with a price of Rs.104.75
and a yield to maturity of 8.79%.

1 Which of these bonds has the greatest reinvestment rate risk?


2 Which of these bonds has the greatest interest rate risk?
3 Which of these bonds has the longest duration?

Solution: Answer is option is A. Reinvestment rate risk is the risk that cash flows that occur during the

Investment Planning – 19Oct 07 29


holding period of the investment will not be able to be reinvested at a rate of return at least as great as
the internal rate of return (or yield to maturity) of the investment. Since Bond A has the highest coupon
payment (and, therefore, the highest cash flow during the holding period), it has the greatest amount of
reinvestment rate risk.

150. Investment A costs Rs.10,000,000 and offers a single cash inflow of Rs.13,000,000 after 1 year.
Investment B costs Rs.1,000,000 and will be worth Rs.2,000,000 at the end of the year. The
appropriate discount rate or required rate of return is 10% compounded annually. Match the
investment(s) listed below with the corresponding financial information in the items that follow.

A. Investment A.
B. Investment B.
C. Both A and B.
D. Neither A nor B.

1 The net present value (NPV) is Rs.818,182 and the internal rate of return is 30%.
2 The NPV is Rs.818,182 and the internal rate of return is 100%.
3 The NPV is Rs.1,818,182 and the internal rate of return is 30%.

Solution: Answer is option D. Net Present Value (NPV) is defined as the difference between the
discounted inflows and the initial outflow. For example, if the discounted cash inflows were Rs.10 and the
initial outflow was Rs.8, then the NPV would be Rs.2.

Net Present Value


Investment A Investment B
FV = Rs.13,000,000.00 Rs.2,000,000.00
N = 1 1
i (discount rate) = 10% 10%
PV = Rs.11,818,181.82 Rs.1,818,181.82
Less Initial
= (10,000,000.00) (1,000,000.00)
Outflow
NPV = Rs.1,818,181.82 Rs.818,181.82
Internal Rate of Return
Investment A Investment B
FV = Rs.13,000,000.00 Rs.2,000,000.00
N = 1 1
PMT = Rs.0 Rs.0
PV = (Rs.10,000,000.00) (Rs.1,000,000.00)
i = 30% 100%

Investments – Securities (moderate)


151. Which of the following is the correct justification for use of an investment in a client‟s portfolio?

A. FNMA securities because they are backed by the full faith and credit of the
U.S. government.

Investment Planning – 19Oct 07 30


B. Call option purchases because they are a low-cost method of hedging a portfolio.
C. Blue chip common stocks because they provide a hedge against inflation.
D. Aggressive growth stocks because they perform better during economic contractions.

Solution: Answer is option C; blue chip common stocks provide a good hedge against inflation, because
they typically provide a return that significantly outpaces inflation. A is incorrect because the government
does not back FNMA securities. B is incorrect because Call options purchased by a client will not help
protect a client from a downturn in the market. D is incorrect because aggressive growth stocks generally
perform worse during economic contractions than less aggressive investments.

Investments – Security Valuation (easy)


152. The Zeta Corporation‟s current dividend is Rs.3.85. If future dividends are expected to grow at
4% forever, which of the following amounts should Zeta stock sell for if the required rate of return on the
stock is 14%?

A. Rs.28.57.
B. Rs.38.50.
C. Rs.40.04.
D. Rs.41.60.

Solution: Answer is option C. This is a question regarding the constant dividend growth model for
determining the value of a stock. The following formula is used for the constant dividend growth model:
D1P0 = where:
k-g

P0 = Price for the security.D1 = The dividend paid at period 1.


k = The investor‟s required rate of return.
G = The growth rate of the dividends.
Therefore, the value of the stock is Rs.40.04, calculated as follows:
(Rs.3.85)(1.04)
P0 = = Rs.40.04 .14 - .04

Investments - Bonds (moderate)


153.Treasury zero-coupon bonds are particularly suited to which of the following types of accounts?
A. IRA.
B. Trust.
C. Corporate.
D. Joint.

Solution: Answer is option A. Treasury zero-coupon bonds do not pay interest to the investor. However,
the accrued interest is taxable each year. Therefore, this type of investment is best suited for a tax-
advantaged account.

Investments - Securities (moderate)


154. A client has a growth objective but requires a large percentage of the return to be tax efficient.
Which of the following products would be most appropriate for this client?

A. Nonleveraged equipment leasing.


B. Balanced mutual fund.
C. Preferred stock mutual fund.
D. Stock index fund.

Solution: Answer is option D.A stock index fund is a mutual fund that mirrors a stock index. Typically, the
fund pays little or no dividends, and is therefore tax efficient. A is incorrect, nonleveraged equipment
leasing investments have more of an income objective than a growth objective and are not tax efficient
because the income is taxed each year. B is incorrect because A balanced mutual fund usually invests a
large percentage of the fund assets in fixed-income securities. C is incorrect because preferred stocks
often pay a large dividend and usually have an income objective.

Source INV Plng 24 Q Word Doc

Investment Planning – 19Oct 07 31


155. Rani invested Rs.10,000 in a fixed deposit. If the interest is compounded monthly at an annual rate
of 4%, what would be the amount that Ali would receive in five years time? (use 2 decimal places)

a. 12,158.65
b. 12,185.65
c. 12,188.65
d. 12.187.65

156. Rekha has borrowed Rs.2,500 at 2% annual interest rate compounded quarterly. What is the
amount she has to repay after five years?

e. 2,762.24
f. 2,763.24
g. 2,769.24
h. 2,768.28

157. Avni wants to accumulate Rs.15,000 in three years time for a one-month USA trip upon her
graduation. Assuming she can get 6% annual return compounded semi-annually from her investment,
how much must she invest today to achieve her goal?

a. 12561.26
b. 12562.11
c. 12562.26
d. 12564.26

158. Chetan borrowed Rs.5,000 five years ago. Assuming that he has to repay Rs.6,500 now, how
much interest rate was he charged?

5.28
5.39
5.93
5.82

159. A bond of face value of Rs. 1000 has a coupon of 7.5% and is compounded quarterly duration 4 yrs.
similar bond in market yield 8% what is pv of the bond?

a) 982.03
b) 983.03
c) 984.03
d) 985.03

160. Mona invests Rs. 21000 for 3 yrs @ 6 %p.a. compounded annually for first 1 yr and quarterly for
next 2 years. What would be her maturity benefit?

25076.35
25075.72
18951.71
20277.68

161 A bond of face value of Rs. 1000 has a coupon of 7.5% and is compounded semi annually duration
4.5 yrs. similar bond in market yield 8% what is PV of the bond?

981.41
982.42
983.25
980.25

162. Mahesh invests Rs. 18000 for 4 yrs @ 8 % p.a. compounded quarterly for first 2 yrs and annually for
next 2 years. What would be his maturity benefit?

24599.22
24566.33
25501.33

Investment Planning – 19Oct 07 32


26502.23

163. A bond of face value of Rs. 1000 has a coupon of 8.5% and is compounded annually, duration 12
yrs., similar bond in market yield 9% what is PV of the bond?

963.20
964.20
964.80
965.80

164. Mary invests Rs. 15000 for 3 yrs @ 8 % p.a. compounded annually for first 1 yr and half yearly for
next 2 years. What would be her maturity benefit?

18524.21
18951.71
19850.71
19849.21

165.Mr. Singh buys 200 convertible debentures of TISCO at Rs. 200 each. 50% of the value of these
debentures is converted into one share of Rs. 80 each after 4 years. Mr. Singh exercises his options after
4 yrs and receives 100 shares. Compute cost of acquisition of each share?

a) 200
b) 250
c) 275
d) 300

166. Meena invests Rs. 16000 for 4 yrs @ 6 % p.a. compounded annually for first 1 yr and quarterly for
next 3 years. What would be her maturity benefit?

a) 20277.68
b) 20278.22
c) 22285.25
d) 20275.78

167. Mr. Satish buys 300 convertible debentures of ABB at Rs. 300 each. 50% of the value of these
debentures is converted into one share of Rs. 80 each after 5 years. Mr. Singh exercises his options after
5 yrs and receives 120 shares. Compute cost of acquisition of each share?

a) 350
b) 360
c) 375
d) 380

168. A bond of face value of Rs. 1000 has a coupon of 7.5% and is compounded semi annually, duration
17.5 yrs., similar bond in market yield 8% what is PV of the bond?

a) 956.34
b) 953.34
c) 1074.43
d) 987.60

169. Miss Savi buys 250 convertible debentures of Reliance Ind. at Rs. 300 each. 60% of the value of
these debentures is converted into one share of Rs. 80 each after 7 years. Miss. Savi exercises his
options after 7 yrs and receives 100 shares. Compute cost of acquisition of each share?

a) 400
b) 420
c) 425
d) 450

170. A bond of face value of Rs. 1000 has a coupon of 6.5% and is compounded semi annually, duration
13.5 yrs. similar bond in market yield 7% what is PV of the bond?

Investment Planning – 19Oct 07 33


a) 957.79
b) 985.79
c) 956.79
d) 958.79

171. Miss Seema buys 250 convertible debentures of Asian hotels at Rs. 300 each. 60% of the value of
these debentures is converted into one share of Rs. 100 each after 7 years. Miss. Seema exercises his
options after 7 yrs and receives 150 shares. Compute cost of acquisition of each share?

a) 225
b) 250
c) 275
d) 300

172. A bond yield 10% annually with interest paid semi- annually. Duration is 3 years. The yield on
bond is 12%. Find current price.

a) 1000.83
b) 1050.43
c) 986.36
d) 950.83

173. Mrs. Shikha buys 250 convertible debentures of ACC. at Rs. 300 each. 60% of the value of these
debentures is converted into one share of Rs. 50 each after 8 years. Mrs. Shikha exercises his options
after 8 yrs and receives 200 shares. Compute cost of acquisition of each share?

200
225
250
275

174. A bond of face value of Rs. 1000 has a coupon of 7.5% and is compounded quarterly, duration
16 yrs. similar bond in market yield 8.5% what is PV of the bond?

912.98
912.38
913.28
914.88

175. Ravi invests Rs.15000 @12% compounded monthly .What will be the value of his investment after 6
years?

31706.49
30706.49
30705.29
30704.29

176. Mr. Sushil buys 300 convertible debentures of Telco at Rs. 250 each. 40% of the value of these
debentures is converted into one share of Rs. 50 each after 6 years. Mr. Sushil exercises his options after
6 yrs and receives 80 shares. Compute cost of acquisition of each share?

375
380
385
390

177. A bond of face value of Rs. 1000 has a coupon of 7.5% and is compounded semi annually duration
5 yrs. similar bond in market yield 8% what is PV of the bond?

978.62
Investment Planning – 19Oct 07 34
978.42
978.52
979.72

178. Reshma invests Rs. 5000 @8% compounded monthly .What will be the value of his investment after
4 years?

6877.33
6876.33
6878.33
6876.88

Source: C:\WINDOWS\TEMP\quiz-apr-inv1.html
179. The best method of valuing a share is:

a) Book value based on net tangible assets.


b) Liquidation value based on the proceeds of liquidation of the company.
c) Present value of all the dividends to be received from holding that share.
d) Apply the P / E ratio to expected earnings per share.

180. If a bond is selling at a premium ______________________________

a) It is an attractive investment
b) Its realised compound yield will be less than the yield to maturity
c) Its coupon rate is below market rate
d) Its current yield is lower than the coupon rate

181. Which of the following is NOT a characteristic of a balance fund?

a) It provides both growth and income objectives


b) It is less risky than growth funds
c) It is more risky than income funds
d) It must invest in both equity and bonds in equal amount

182. Purohit has just bought a house for Rs.8,00,000. He wants to sell it in 20 years time for Rs.
20,00,000. His friend who is a real estate agent estimates that the house will appreciate in value by 6%
for the first 8 years, 5% for the next 8 years and 4.50% for the last four years. Will will be the value of the
house after 20 years and will Purohit be able to realise his desired value ?

a) 21,85,489, Yes
b) 22,46,552, Yes
c) 19,54,413, No
d) 17,45,387, No

183. Disclosure statements to prospective clients include all of the following except:

a) Performance record of other clients


b) The method of remuneration, fees and commissions
c) Access to internal and external complaint handling mechanism
d) Disclosure of any conflict of interest

184. Which of the following costs best describes the cost of foregone income that results from making an
economic decision to use funds to purchase a piece of equipment?

a) Cost of Capital
b) Fixed Cost
c) Marginal Cost
d) Opportunity Cost

185. A mutual fund that invests in Indian Equities, foreign equities, Indian Corporate Bonds, Indian
Government Gilts is subject to the following risks?

Investment Planning – 19Oct 07 35


1.Business Risk,
2.Default Risk,
3. Systematic Risk,
4. Interest Rate Risk.

a) 1 & 3 only
b) 1,3 & 4 only
c) 3 & 4 only
d) 1,2,3 & 4

186. Which of the following statements concerning technical stock market indicators is/are correct?

1. The stock market is considered strong when the volume of the market is increasing in a rising market.
2. The market's direction will change when the percent of odd-lot short sales significantly increases
or decreases.
3. Prices crossing the moving average line would be an indication of the change in the market.

a) 1 only
b) 1 & 2 only
c) 2 & 3 only
d) 1,2 & 3

187. If a new issue was offered to the public at 15 times earnings but the market was pricing similar
shares at 19 times, this would be _____________.

a) Appalling proposition to the investor


b) The investor cannot take a position
c) An example of low gearing
d) Bargain not to be missed

188. Mr. X buys 50 TISCO October Rs. 350 call options for Rs. 15. The current share price is Rs. 345.
The break-even share price, ignoring transaction costs is Rs.________.

a) 350
b) 360
c) 365
d) None of the above

189. Investor complaints relating to the following Capital Market issues will not be entertained by SEBI:

a) A company declaring no dividend on equity for the fourth consecutive year.


b) A company has declared dividend but not paid the same after six months of declaration.
c) A company not paying the redemption proceeds on debentures issued by the company,
one year after maturity date.
d) None of the above cases.

190. If a bond is selling at a premium:

a) It is an attractive investment
b) Its coupon rate is below market rate
c) Its current yields is lower than the coupon rate
d) Its realized compound yield will be less than the yields to maturity

191. In ranking portfolio performance, which measure of risk does the Treynor Index use?

a) standard deviation
b) variance
c) beta
d) alpha
Investment Planning – 19Oct 07 36
Solution: By definition, the Treynor Index uses beta as its risk measure.

192. A portfolio manager can hedge a share portfolio by_______.

a) Buying call options


b) Selling call options
c) Buying index options
d) Selling put options

193. The trust deed lays down the terms and conditions under which the unit holders money is to be
invested. Specifically, it details:

a) The unit trust scheme


b) The types of authorized investments
c) All fees and charges
d) All of the above

194. Beta is a measure of:

a) Geometric average return


b) Holding period return
c) Systematic risk
d) Unsystematic risk

195. If the current share price is S and the set exercise price is X, the intrinsic value of the call option
is_______.

a) Max (O, S-X)


b) Max (O, X-S)
c) Min (O, S-X)
d) Min (O, X-S)

196. The CAPM is a model that:

a) Determines the geometric return of a security.


b) Determines time-weighted return
c) Explain return in terms of risk.
d) Explains systematic risk

197. GE is an AAA rated issuer of Corporate Bonds in the International Debt markets. The issue price of
a typical GE corporate bond is affected by all the following EXCEPT the _______.

a) Face value, coupon rate, and maturity of the bond.


b) Firms required return on debt.
c) Percentage of debt in the firm's capital structure.
d) Required return on the firm's competitors' bonds.

198. Public Issue through the book building process is better than I.P.O at fixed price because_______.

a) High fixed price will result in under subscription leading to loss to the investor.
b) It helps the issuer to ascertain the exact price at which the investor is willing to
subscribe.
c) Low fixed price will result in over subscription leading to loss to the issuer.
d) All of the above

199. A client purchased a mutual fund with a Rs 10, 000 lump-sum amount four years ago. During the
four years, Rs 4, 000 of dividends was reinvested. Today the shares are valued at Rs 20,000 (including
any shares purchased with dividends). If the client sells shares equal to Rs13, 000, which statement (s)
is/are correct?
1. The taxable gain can be based on an average cost per share.

Investment Planning – 19Oct 07 37


2. The client can choose which shares to sell, thereby controlling the
taxable gain.
3. To minimize the taxable gain today; the client would sell shares with the
higher cost basis.
4. The client will not have a gain as long as he/she sells less than what
he/she invested.
a) 1, 2, and 3 only.
b) 1 and 3 only.
c) 2 and 4 only.
d) only.

200. The best method of valuing a share is:

a) Book value based on net tangible assets.


b) Liquidation value based on the proceeds of liquidation of the company.
c) Present value of all the dividends to be received from holding that share.
d) Apply the P / E ratio to expected earnings per share.

201. Which combination of the following statements about investment risk is correct?
1. Beta is a measure of systematic, non-diversifiable risk.
2. Rational investors will form portfolios and eliminate systematic risk.
3. Rational investors will form portfolios and eliminate unsystematic risk.
4. Systematic risk is the relevant risk for a well-diversified portfolio.
5. Beta captures all the risk inherent in an individual security.

A. 1, 2, and 5.
B. 1, 3, and 4.
C. 2 and 5.
D. 2, 3, and 4.
E. 2 and 5.

202. Which of the following asset allocations would you recommend to a 60 year old retiree who depends
on his investments for monthly income?

Fixed Deposits: 60% Properties: 30% Equities: 10%


Fixed Deposits: 40% Properties: 30% Equities: 30%
Fixed Deposits: 20% Properties: 40% Equities: 40%
Fixed Deposits: 10% Properties: 40% Equities: 50%

Solution: A retiree at 60 has little appetite for risk as he no longer has any earning power. Further, he
needs certainty of income. But a small amount of equity is still recommended as a hedge against inflation.
If not, if he lives until say 80, he will experience a drop in his standard of living.

203. The following are the long-term credit ratings given by CRISIL, Which is the lowest investment grade
category?

AAA
AAA+
BBB+
BBB-

204. Rank the following funds in the order of increasing risk:

I Growth Fund, II Balanced Fund, III Bond Fund , IV Small Cap Fund

a) III, IV, II, I


b) III, II, IV, I
c) III, II, I, IV
d) III, IV, I, II

Investment Planning – 19Oct 07 38


Solution: It is generally accepted that the order is Bond Fund, Balanced Fund, Growth Fund and Small
Cap Fund. A small cap fund is more risky than a growth fund. Earnings of smaller companies are more
exposed to the vagaries of the economic cycle.

205. The Fund Manager for a particular scheme is an employee of the _______

a) Sponsor
b) AMC
c) Trustee
d) Custodian

206. Capital allocation forms a critical part of _____________

a) Cash Flow Planning


b) Investment Planning
c) Will Planning
d) Insurance planning

207. Implied volatility is a term commonly used in the __________

a) Derivatives market
b) Real Estate market
c) Fixed Income Market
d) Stockmarket

208. Studies show that the Indian middle class has an excess of ______ as investments.

a) Fixed Deposits
b) Mutual Funds
c) Equity Shares
d) Real Estate

209. Only those equity transactions where ______ is paid are eligible for zero long term capital gains tax

a) Service tax
b) Securities Transaction Tax
c) Brokerage
d) Depository Participant Charges

210. ______________ funds have the highest expense ratio

a) Customised
b) Novated
c) Standardised
d) Without counterparty risk

211. Purchasing a Futures Contract __________

a) Confers a right but no obligation on the buyer


b) Confers an obligation but no right to the buyer
c) Enables an investor to take a leveraged position
d) Protects the buyer against downside risk

212. ______________ equity funds are appropriate for an Indian investor with a penchant for global
diversification

a) Diversified
b) Sector
c) International
d) Global

Investment Planning – 19Oct 07 39


213. Your liquid fund investment is treated as a ____________ asset

a) Intangible
b) Investment
c) Near-Cash
d) None of the above

214. Which of the following factors will result in an increase in the duration of corporate bond.

 I an increase in the number of years to maturity.


 II a decrease in the coupon rate
 III change from annual to semiannual coupon payment.
 IV change from annual coupon to zero coupon bond

a) I, II & III only


b) I, II and IV only
c) I, III & IV only
d) II, III & IV only

Answer: A change in the coupon payment from annual to semiannual will result in the coupons
becoming more significant compared to the final payment on a weighted average basis and decreases
the duration of a bond . All other factors will increase the duration of a bond.

215. A lump sum investment option in a mutual fund will be most warding in __________________

a) A secular uptrend
b) A secular downtrend
c) Volatile markets
d) A sideways market

216. Tata Power is raising funds through a bond issuance to fund a new power plant at Noida, UP. They
are issuing Two Year maturity, Zero-coupon bond with face value of Rs 1000 and yield of 4%. What price
would you pay for this Tata Power Zero-coupon bond today?

a) Rs. 920.00.
b) Rs. 924.56.
c) Rs. 925.95.
d) Rs. 960.00.

217. Equal amount of investment is made in portfolio consisting of securities X and Y. Standard deviation
of X is 12.43%. ; Standard deviation of Y is 16.54%. ; Correlation coefficient is 0.82. ; The interactive risk
of the portfolio, measured by covariance is

a) 145.64
b) 156.22
c) 168.59
d) 172.56

218.The risk free return of Security A is 8%. In addition to it, you expect that the return on market would
be 14%. The expected return of Security A with beta of 0.70 is ________.

a) 12.2%.
b) 15.4%.
c) 17.8%.
d) 18.2%.

219. The debt – asset ratio is a useful tool to primarily measure ______________

Liquidity
Investment Planning – 19Oct 07 40
Solvency
Savings potential
Debt servicing capacity

220. Portfolio A had a return of 12% in the previous year, while the market had an average
return of 10%. The standard deviation of the portfolio was calculated to be 20%, while the standard
deviation of the market was 15% over the same time period. If the correlation between the portfolio and
the market is 0.8, what is the Beta of the portfolio A?

a) 0.94
b) 1.07
c) 1.31
d) 1.91

Solution: Beta=[(S.D of security / S.D of Market) *Coefficient of correlation]

221. The following are statements made concerning contracts of insurance. Identify the statement/s that
is/are correct.

I. For life insurance contracts, misstatement of the insured‟s age constitutes avoidable misrepresentation.
II. An innocent misrepresentation by an applicant for insurance constitutes fraud.

A. I only
B. II only
C. I and II
D. Neither I nor II

222. Which of the following statements concerning the movement in foreign exchange rates are true,
assuming that all other factors remain unchanged?

 I The exchange rate will appreciate with an increase in domestic interest rates
 II Increasing international reserves leads to a stronger exchange rate
 III Strengthening the exchange rate leads to a temporary increase in the competitiveness of
exports
 IV Lower domestic inflation leads to a weakened exchange rate

a) I & II only
b) II & III only
c) III & IV only
d) I & IV only

Solution:
III A strengthening exchange rate will lead to a drop in competitiveness of a countries exports – statement
in false. IV Statement in false. Lower domestic inflation will lead a strengthening exchange rate. The other
statements are true.

223. A corporation proposes to issue a 7-year bond with a coupon rate of 8.0%. The prevailing yield to
maturity of bonds with similar risk and term is 10.0%.The bond will sell at ____________ to its face value.

a) a premium
b) a discount
c) par
d) a predetermined price

Solution: Given that the coupon rate is less than the yield to maturity of similar instruments, the bond will
sell at a discount.
Investment Planning – 19Oct 07 41
224. Calculate the net price earnings ratio (PER) of the stock of Company A, with the following
information:

 Price = Rs.5.00
 Profit before tax = Rs.75.0 million
 Profit after tax = Rs.54.0 milllion
 Paid up Capital = Rs.100 million at par value of Rs.0.50 per share

a) 6.7
b) 9.3
c) 13.3
d) 18.5

Solution: The calculations are as follows: EPS = 54/200 = Rs.0.27, PER = 5.00/0.27 = 18.5

225. Mr A‟s stock market portfolio consistently outperformed the BSE Sensex based on tips he received
from a friend working in a corporate finance department. Which of the following is his success a violation
of?

a) weak form of market efficiency


b) semi-strong form of market efficiency
c) strong form of market efficiency
d) total market efficiency

Solution: The strong form of market efficiency maintains that prices contain all information, private and
public. The fact that Mr A was able to beat the BSE is a violation of the strong form of market efficiency

226. Which of the following have a negative impact on stock prices?

 I increase in risk premiums


 II increase in dividend growth rate
 III increase in the discount rate
 IV increase in interest rates

a) I II & III only


b) I II & IV only
c) I, III & IV only
d) II, III & IV only

Solution: The only answer that does not contain II. An increase in the dividend growth rate leads to an
increase in the stock price

227. What is the expected return of an investment with the following distribution of possible outcomes?

RETURN (%) Probability


7 20%
12 30%
20 15%
5 35%

Total 100%

a) 25.00
b) 11.00
c) 9.75
Investment Planning – 19Oct 07 42
d) 9.50

Solution: (7x0.2) + (12x0.3) + (20x0.15) + (5x0.35) = 9.75

228. Stock A and stock B are positively correlated with a correlation coefficient of 0.75. When stock A
moves up by 12%, how will stock B perform?

a) Stock B will move up by 12%


b) Stock B will move down by 12%
c) Stock B will move up by 9%
d) Stock B will move down by 9.0%

Solution: By definition, as the stocks are positively correlated, they move in the same direction but Stock
B will move three quarter as much as Stock A.

229. Which of the following statements regarding investment risk are true?
 I Beta captures all the inherent risks in an individual security
 II Unsystematic risk is reduced in a portfolio because securities are not perfectly
correlated
 III As Beta increases, the expected return also increases
 IV Rational investors will form portfolios to eliminate systematic risk

a) I & II only
b) II & III only
c) III & IV only
d) I & IV only

Solution: Beta only captures systematic non-diversifiable risk of a security. Therefore, I False
II True, III True, IV False. Investors form portfolios to eliminate non-systematic risk

230. Which of the following are among the advantages of money market funds?

 I Provides current income


 II Provides safety of principal
 III Notice is required for withdrawal
 IV Provides capital gains

a) I only
b) I & II only
c) I, II and III only
d) All of the above

Solution: A money market fund invests only in short term money market instruments such as short term
debt securities, Treasury bills, banker‟s certificates of deposits and bank acceptances. As the instruments
are highly liquid, usually no notice for withdrawal is required. In fact, some money market funds provide
quasi checking facilities. Money market funds can pay current income as the instruments mature in the
short term. It should also be mentioned that money market funds have low interest rate risk because of
the short tenor of the instruments.

231. Which of the following are not characteristics of a closed end fund?

I Offers a fixed number of units at the time of the initial public offering
II The manager is obligated to repurchase units on request
III Typically trades at a discount to its net asset value
IV The fund will have to reduce equity allocation in the face of redemptions

a) I & III only


b) I & IV only
c) II & III only
Investment Planning – 19Oct 07 43
d) II & IV only

Solution: Only I and III are true. II There is no need for the manager to repurchase units. IV Redemptions
do not affect a close end fund. The number of units stays the same.

232. Mr A is 30 years of age and intends to retire when he has Rs.400,000. He believes that he can save
Rs.10,000 at the end of each year. Being risk adverse, he places his funds in fixed deposits earning only
5.0% pa. His savings at this point is Rs.20,000. At what age will he be able to retire?

a) 53 years
b) 51 years
c) 68 years
d) 70 years

Solution: PV is –Rs.20,000, FV is 400,000 with yearly payments of – Rs.10,000. Number years is 21,
implying retirement age of 51.

233. What is the net present value (NPV) of a Rs.8.0 million investment with the following cash flows:
(Rs. millions)
Year 1 Year 2 Year 3
2.0 5.0 6.0
Assume a discount rate of 10% pa for all years.

a) Rs.2.5 million
b) Rs.3.8 million
c) Rs.5.0 million
d) Rs.10.5 million

Solution: Calculations are as follows: (2.0)/(1.1) + (5.0)/(1.1)^2 + (6.0)/(1.1)^3 – 8.0

234. Mr A places his fixed deposit for one year with interest payable at maturity. Mr B places his fixed
deposit with another bank with interest payable quarterly. Assuming that the deposit amount is
Rs.100,000, and using the same annual rate of 6%, what is the difference in future value after one year?

a) Rs.130.43
b) Rs.132.54
c) Rs.134.22
d) Rs.136.36

Solution: (100,000) x (1 + 0.015)^(4) – (100,000 x 1.06) = 136.36

235. Which of the following assumptions support the use of technical analysis?

a) Future performance should be reflective of past performance


b) The values of market indices and stock prices are determined based on supply and
demand
c) Stock prices move in trends that would persist over long periods
d) All the above

236. Of the four pairs of portfolios, which pair provides the highest level of diversification?

a) Portfolio 1 & 2: with a correlation coefficient of +0.92


b) Portfolio 3 & 4: with a correlation coefficient of +0.37
c) Portfolio 5 & 6: with a correlation coefficient of 0
d) Portfolio 7 & 8: with a correlation coefficient of -0.78

237. The profit earned by a Mutual Fund will be directly proportional to its

The level of the BSE Sensex


Assets under Management
Investment Planning – 19Oct 07 44
Its operating expenses
The sponsor‟s net worth

238. Anil Kumar has two Mortgage Loan options before him. The interest rate, and other conditions are
the same for both, except that one has a repayment term of 15 years and the other has a repayment term
of 30 years. Anil wants to evaluate the EMIs for both terms. All other conditions being the same,
repaying a loan in 15 years instead of 30 would require EMIs that are________.

Half the size of the 30-year loan payments.


Less than twice as large as the 30 year loan payments.
More than twice as large as the 30 year loan payments.
Twice as large as the 30 year loan payments.

Solution:
Suppose a loan of Rs 1,00,000
Interest = 8%
EMI for 15 Years=Rs 11682.95
EMI for 30 Years=Rs 8882.74
Rs 11682.95 < 2 * 8882.74

239. You are running a Dividend Yield Fund for a leading Mutual Fund House. The most recent dividend
of All Is Fine Business Services common stock was Rs 2.35. The dividends are expected to grow at 4
percent indefinitely. If you are looking at a 12 percent return, how much will you be willing to pay for one
share of All Is Fine Business Services?

a) Rs. 24.79.
b) Rs. 29.38.
c) Rs. 30.55.
d) Rs. 32.45.

Solution: M.V of a Stock=[(Dividend*(1+growth rate)] / (Return- growth rate)

240. You are an owner of an apartment complex with 300 units, each of which can fetch Rs. 1000 p.m. as
rentals. The apartment complex has an average occupancy rate of 75%. The expenses for maintaining,
up keeping the apartment comes to around Rs. 10 Lakh p.a. Based on the concept of capitalized earning
approach and assuming that you require a capitalization rate of 10%, how much is the complex worth
now?
a) Rs. 1.50 crore
b) Rs. 1.70 crore
c) Rs. 2.00 crore
d) None of the above.
Solution:
Annual rent receivable =300*(1000*12)*0.75 (A)
Annual upkeep & maintenance =Rs 10 lakhs (B)
Net income annually=Rs 17,00,000 (A-B)
Capitalization rate=10%
Value of the complex=Net income / Capitalization rate

241. Given the following information about securities A and B:

Historical Returns for Securities


A B
Year 1 10% 18%
Year 2 6% 12%
Year 3 0% 2%

242. Which of the following are true about Securities A and B?

1) A is more risky because it has a higher standard deviation.


2) B is more risky because it has a higher standard deviation.
3) A has a higher risk-adjusted return.
4) B has a higher risk-adjusted return.

Investment Planning – 19Oct 07 45


a) 1 and 3
b) 1 and 4
c) 2 and 3
d) 2 and 4

243. Given the capitalization rate of the following link houses in the BKC area, how much should your
client pay for the link house in BKC 4?

Comparable Annual Net Market Value Market


Property Income (Rs.) (Rs.) Capitalization
(1000 sq ft) Rate (%)
BKC 1 14,400 342, 800 4.20%
BKC 2 16,800 308,200 5.45%
BKC 3 12,600 210,000 6.00%
BKC 4 15600 ?

a) Rs. 306,657
b) Rs. 303,121
c) Rs. 299,041
d) Rs. 293,786

Solution: Average market capitalization rate: (4.20% + 5.45% + 6.00%)/3 = 5.22% Market value: Rs.
15,600/0.0522 = Rs.299,041

244. Calculate the yield to maturity of a bond with the following parameters:
Face Value : Rs.1000.00
Market Price : Rs.1040.00
Coupon Rate (paid annually) : 7.0%
Remaining Term to Maturity : 5 years

a) 8.10%
b) 7.69%
c) 7.00%
d) 6.05%

245. Using the Growth Dividend Model, calculate the price of the stock of Company A, with the following
information:

 EPS =Rs.10 per share


 Current Dividend = Rs.8 per share
 Dividend growth rate = 5.0%
 Risk free rate = 6.0%
 Company A risk premium = 7.0%

a) Rs.114
b) Rs.105
c) Rs.100
d) Rs. 62

Solution
Using the Growth Dividend Model, the calculations are as follows:

R = Risk free rate + risk premium


Div1 = Div x (1 +g)

Investment Planning – 19Oct 07 46


P = Div1/(R – g)

Where P = price
Div1 = dividend in year 1
R = required rate of return or discount rate
g = constant dividend growth rate
R = 0.06 + 0.07 = 0.13
Div1 = 8 x (1 + 0.05) = 8.4
P = 8.4/(0.13 – 0.05) = 105

246. An investor invested Rs.4000 in each of Funds A & B with details as provided below. Both funds
subsequently appreciated by 18% based on their NAV.Which of the following statements is/are true?

Buying Selling NAV


Fund A 0.96 1.02 0.95
Fund B 0.55 0.58 0.55

 I The investor obtained 3,921.57 units in Fund A.


 II The investor obtained 6,896.55 units in Fund B.
 III The investor would have achieved a higher return by investing in Fund A alone
 IV The investor would have achieved a higher return by investing in Fund B Alone

a) I & II only
b) I, II & III only
c) I, II & IV only
d) III only

Solution: As both funds appreciated by the same amount, the investor made identical returns from both
funds: Rs.4000 x 18% = Rs.720. There is a misconception that a lower priced fund has a higher return.
I is true. Rs.5000/1.02 = 4,901.96 units II is true. Rs.5000/0.58 = 8,620.69 units

247. What is the duration of a zero coupon bond with yield to maturity of 6% maturing in 6 years time?

4.35
5.34
6.00
6.35

248. Calculate the return of an investor who bought units of Fund A on 31/12/06 and sold the units on
31/12/07 with the information provided below?

Fund A BUY SELL NAV


Price (31/12/06) : Rs106 113 106
Price (31/12/07) : Rs.112 118 112
Dividend paid in 2007 : Rs. 6

a) 0.9%
b) 4.4%
c) 11.3%
d) 17.0%

Solution:
Total return is calculated as follows: (P2 – P1 + Div)/P1 x 100, (112 – 113 + 6)/113 = 4.4%

249. Bank A quotes the following fixed deposit rates:

1 month 5.3%
months 5.6%
months 5.8%
12 months 6.2%

Investment Planning – 19Oct 07 47


250. Mrs K expects the six-month fixed deposit rate to rise in six months time. She wants to know above
what level will the six month rate have to rise in six months time, for her to be worse off, in placing her
funds for one year today?

a) 6.0%
b) 6.2%
c) 6.4%
d) 6.6%

251. The most appropriate investment for a highly risk averse investor aged 57 is __________
a) An index fund based on the Nifty.
b) A diversified equity fund
c) A Maharashtra State Government Bond
d) A SBI Fixed Deposit

252. In industry life cycle, revenue, margin and profit are expected to peak in the following order:

a) Revenue, margin, profit


b) Margin, profit, revenue
c) Margin, revenue, profit
d) Profit, margin, revenue

253. The broadest measure of inflation is:

a) Consumer Price Index


b) Wholesale Price Index
c) Both a and b

C:\WINDOWS\TEMP\quiz-apr-inv2.html

254. NPV is calculated in the case of a series of ---------------- cash flows.

a) Zero
b) Single
c) Uneven
d) Even

255. The Present Value of a sum of money _________ as the Discounting Rate ______________.

b) Remains same, Increases


c) Decreases, Decreases
d) Increases, Increases
e) Increases, Decreases
f) Data Insufficient

256. Classifying an investment as a long term investment depends primarily on;

a) the length of time the investor expects to hold the investment.


b) the amount of the investment.
c) whether a liquid market exists for selling the investment.

257. A period when an economy is experiencing substantial growth and either a declining jobless rate is
called_____________.

a) Stagflation
b) Deflation
c) Depression
d) Boom

Investment Planning – 19Oct 07 48


258.In India Futures contracts in ___________ may be settled by delivery.

a) Commodities
b) Stocks
c) Stock Index

259. Money has time value. It derives this value due to existence of several conditions. Which one of the
following is not one of the conditions contributing to the existence of this value?

a) The fees and commission sources of the firm


b) Possibility of increase in tax rates over time.
c) Ability to buy/ rent assets generating revenue
d) Cost of foregoing present consumptions

260. You have term deposits of Rs. 4,00,000 with a bank. In order to meet sudden requirements for
liquidity and short-term credit, you are applying for an overdraft facility with the bank. What is the rate of
interest you will pay on this facility?

a) The bank will apply a flat rate of interest on the amount of overdraft allowed to actually utilize.
b) The bank will apply a flat rate of interest on the amount of overdraft allowed to you.
c) The bank will apply rate of interest linked to the term deposit rate, on the amount of overdraft
utilized.
d) The bank will apply rate of interest linked to the term deposit rate, on the average amount of
overdraft remaining

261. The Nifty has doubled since the last time you advised your client to reduce his equity exposure. The
client is annoyed. What might be the most appropriate action to take immediately?

a) Apologize for wrongly forecasting the market


b) Change his asset allocation by increasing his equity exposure
c) Help the client understand the logic of his asset allocation
d) Rebalance his asset allocation by reducing equity investments

262. A professional indemnity policy protects the insured from risk arising out of _____________.

a) Intentional misconduct
b) Misrepresentation of professional competence
c) Negligence
d) Undisclosed conflict of interest

263. Protector International is a financial services firm that specializes in investment advisory services. In
its brochure for Financial Planning services, it may state _____________.

a) It can offer superior investment returns on customer portfolios and talk of the arrangements to
offer advice in other areas
b) It has the competence to take care of all Financial advisory requirements of the customer
c) Its competence in investment advisory services and the arrangements to offer advice
in other areas
d) Its Financial Planning services are the best available in the market in light of its investment
advisory capabilities and arrangements to offer advice in other areas

264. Which of the following is a concurrent indicator of the phase of the business cycle?

a) Wholesale price Index


b) Index of Industrial production
c) Labor costs and capacity utilization
d) Order levels in the manufacturing sector

265. The effective interest rate earned per rupee _______ as the periods of compounding increase.

a) Increases
b) Decreases
c) Remains same
Investment Planning – 19Oct 07 49
d) Decreases for some time and then increases
e) Data insufficient

266. What is the main difference between the personal Financial Planning needs of the employed
and the self-employed?

a) Attitude to risk/Risk appetite


b) Need to fund children's education
c) Need to fund retirement
d) The extent of any employer-provided pension benefits

267. Immunization protects bondholders from which of the following risk/s:

1) Interest rate risk 2) Reinvestment rate risk 3) Maturity risk

a) 1 only
b) 2 only
c) 1 & 2 only
d) 1, 2 & 3

268. The economy is going through a phase of expansion and growth. Industrial production and
profitability are high. Your client has a portfolio that is heavily invested in bonds. Which of the following
fears of the client is well founded?

a) Higher rates of growth will increase demand for funds and interest rates will firm up,
leading to fall in bond prices.
b) Higher rates of growth will require higher imports and expenses. The government deficits will
go up.
c) The central bank will try to reduce rates to make funding of business cheaper and reduce
costs. That will
d) The currency will become convertible and interest rates will rise as a consequence.

269. Ram is a Financial Planner in a large firm. His wife has some large investments in the shares of a
few companies. Ram is required to offer views on almost all of these holdings to clients. Under the Code
of Ethics and Rules of Professional Conduct _______

a) Ram must disclose the fact to his client(s) so as to make them aware of any potential
conflict of interest
b) Ram has to disclose these holdings only to his employers, if required by the firm's internal
compliance rules
c) Ram need not follow any code of ethics and rules of professional conduct.
d) Ram will not violate the Code and the Rules if he does not disclose his wife's holdings

270. How are financing costs included in NPV and IRR calculations?

a) By including them in the interest payments.


b) By considering the interest rate in the setting of the discount rate
c) As a tax deduction
d) By including them in the earnings

271. According to fundamental analysis, which phrase best defines the intrinsic value of a share of
common stock?

a) The par value of the common stock.


b) The book value of the common stock.
c) The liquidating value of the firm on a per share basis.
d) The discounted value of all future dividends.

272. The term .Efficient Frontier. is contained in________.

a) Technical Analysis
b) Modern Portfolio Theory
c) Value Investing Theory
Investment Planning – 19Oct 07 50
273. A major difference between load and no-load funds is

a) Marketability: no-load funds can be traded more readily.


b) Acquisition cost: load funds cost more than their NAVs.
c) Performance: load funds do better.

274. The Reliance fund trades on the NSE. Its recent price is Rs.10, but its NAV is Rs.12. We know then

a) The fund is closed-end, selling at a discount.


b) The fund is open-end, selling at a premium.
c) The fund is closed-end, selling at a premium.

275. If your investment goal is simply to match the market, should buy a(n)

a) Growth Fund
b) Money market fund.
c) Index fund.

276. You are evaluating a fund. What activity would you typically not undertake in this effort?

a) Calculate or find the fund's rate of return.


b) Calculate the fund's NAV.
c) Find the fund‟s turnover ratio and administrative expenses-to-assets ratio.

277. The Cholamandalam Fund‟s rate of return was 9%, while the market return was 15%.
Cholamandalam‟s beta was 0.5.

a) Cholamandalam's management out-performed the market on a risk-adjusted basis.


b) Cholamandalam's RAROR was 11.5%.
c) Cholamandalam's management under-performed the market on a risk-adjusted basis.

278. You are considering taking a passenger with you when you go home over Christmas break. She
lives 100 kms out of your way, and the total trip is 500 kms. She has offered Rs.50o for the service. You
estimate the total cost of the trip at Rs.300o. You should

a) Reject the offer since 20% (1000/5000) of Rs.3000 is greater than Rs.500.
b) Accept the offer if the opportunity costs of the trip is greater than Rs.2500.
c) Accept the offers if marginal costs associated with 100 kms are less than Rs.500.

279. liquidity ratio of 2.0 tells us that the family has

a) Rs.2 in liquid assets for each Rs.1 in total liabilities.


b) Rs.2 in liquid assets for each Rs.1 in current liabilities.
c) Rs.2 in liquid assets for each Rs.1 of total expenses.

280. In India, Preference shares may be issued for a maximum number of ___________ years.

a) 12
b) 15
c) 10
d) 20

281. Determination of Residential status for the tax purpose is applicable on:

i. Individual ii. Firm iii.Company iv. HUF

a) i,ii,
b) i,ii, iii
c) i only
d) i, ii, iii, iv

282. A growth-oriented non-divided paying share is bought for Rs.250 and sold for Rs. 450 after
Investment Planning – 19Oct 07 51
5 years, the compound annual growth rate is:

a) 14.86%
b) 12.47 %
c) 11.50%
d) 10.71%

283. The call option strike price on a share is Rs. 500 and the current share price is Rs. 550. The call
option premium is Rs. 60. The time value of the option is:

a) 60
b) 10
c) 30
d) 15

284. Mr. A deposits Rs. 10,000 in his own PPF account and same amount in his wife.s account.
How much maximum amount can he deposit in his nephew.s name?

a) Rs. 20,000
b) Nil
c) Rs. 70,000
d) Rs. 60,000

285. A Rs.100 par value bond having 10 % coupon rate will mature after 7 years. Find the value of the
bond if the discount rate is 8 %.

a) 109.85
b) 111.41
c) 108.75
d) 110.60

286. Consider a portfolio of two investments viz. A & B. The sum total of volatility of A and B
respectively, represented by standard deviation of the two investments, will be equal to the
volatility of the portfolio as a whole if _________________.

a) A and B have a correlation of Zero


b) A and B have a correlation of 1
c) The portfolio is equally divided between A and B
d) The return on the portfolio is equal to the sum of returns of A and B

287. Which of the following is a correct interpretation of the Rules of Conduct pertaining to the
Ethic of Confidentiality?

a) A Member must when requested by the client, provide to a person authorized by the client, all
original documents prepared or received by the Member in undertaking the advisory task
b) A Member owes to the Member's partners or co-owners a responsibility to act in good faith
(expectations of confidentiality) only while in business together, not thereafter
c) The Member shall maintain the same standards of confidentiality to employers as to
clients
d) Under no circumstance, will any Member divulge any information or knowledge regarding the
FPSB India or its members that they may know or be exposed to

288. Mr. Rajan's investment portfolio comprises Rs.2 lakh in equity, Rs.5 lakh in debt and Rs.1
lakh in his bank current account. Over one year the returns on equity and debt are 5% and 12%. At the
end of the year to maintain his current asset allocation, he needs to _____________.

a) Do nothing.
b) He needs to move Rs, 10000/- from equity and Rs. 60000/- from debt to cash.
c) He needs move Rs.7500/- to equity from debt and Rs. 8750/-to cash from debt
d) He needs to invest Rs. 70000/- in debt and equity.

289. A 10 year 8.0% bond (Face Value- Rs.1000, interest payable semi-annually) maturing 6 years
Investment Planning – 19Oct 07 52
from today is available at a yield to maturity of 6.0%. It is likely to be priced at _______________.

a) Rs. 1100
b) Rs. 1149
c) Rs. 1168
d) Rs. 1498

290. In India, Mutual Funds have recently moved to the concept of ___________AUM calculation.

a) Monthly average
b) Month end
c) Fortnightly average

291. Raykar is an accomplished Financial Planner and is also an expert on derivatives and high yielding
bonds. He understands client requirements well and is able to come up with appropriate portfolio
restructuring ideas for clients. He believes in quickly moving clients from one investment to another
through a dynamic process of research and recommendations. What according to the Rules relating
to the Code of Ethics is the most applicable in this case?

a) He does not violate the Rules if he explains to the client the reasons and is able to
show that the moves are appropriate to the client
b) He does not violate the Rules since he conducts and has access to research and advises on
products relevant to clients based on an understanding of their requirements
c) He does not violate the Rules since he is an acknowledged expert and knows what is best for
his clients
d) He violates the Rules as it amounts to active churning of client portfolios

292. Mrs. & Mr. Arora are aged 55 and 58 years respectively. Both expect to work till they turn 65.
Their only goal is to fund their retirement. Which of the following is likely to be an appropriate asset
allocation strategy for them?

a) 10% sectoral equity, 20% diversified equity, 30% long-term debt, and 40% medium
term debt
b) 20% Sectoral equity, 60% diversified equity, 20% long-term debt
c) 30% Sectoral equity, 30% diversified equity, 40% cash/ liquid investments.
d) 80% long-term debt, 20% medium term debt

293. In analyzing the position of a portfolio in terms of risk/return on the capital market line (CML),
superior performance exists if the fund's position is the CML, inferior performance exists if the fund's
position is the CML, and equilibrium position exists if it is the CML.

a) Above; on; below.


b) Above; below; on.
c) Below; on; above.
d) Below; above; on.

294. Which of the following best describes the investment characteristics of a high-quality long-term
municipal bond?

a) High inflation risk; low default risk.


b) Low inflation risk; high market risk.
c) Low inflation risk; low default risk.
d) High inflation risk; high market risk.

295. An Asset Management Company must have a minimum corpus of Rs.______________ crores.

a) 5
Investment Planning – 19Oct 07 53
b) 15
c) 25
d) 10

296. A 5 year annual annuity has a yield of 6%. What is the duration?

a) 2.88 years
b) 2.55 years
c) 3.16 years
d) 1.35 years

297.The price of Stellar Ltd. is currently Rs.40. The dividend next year is expected to be Rs.4.00.
Required return on the stock is 12%. Find the expected growth rate under the Constant Growth
model.

a) 2.00 %
b) 2.25 %
c) 1.90 %
d) 2.75 %

298. Data on two stocks is given for 2 different scenarios below. Find Beta of both stocks
Market Return Infocomm Ltd. FMCG Ltd.
5% 3% 7%
15% 25% 12%

a) 1.8, 0.60
b) 2.5, 0.75
c) 1.5, 0.25
d) 2.2, 0.5

Solution: Beta of Infocomm is (25-3)/ (15-5) =2.2; Beta of FMCG = (12-7)/ (15-5) = 0.5

299. Data on a mutual fund is given:


Fund Name Mean Return Std. Deviation Beta
A 10% 25% 0.75
Market Index 16% 20% 1.00

The risk free rate is 9 %; Calculate Treynor, Sharpe and Jensen measures.

a) 1.05, 0.10, 4.25


b) 1.33, 0.04, -4.25
c) 1.10, 0.15, -3.75
d) 1.46, 0.09, 3.75

Solution: Working Note: Treynor Ratio = (Rp-Rf)/ Beta; Hence 1.33; Sharpe Ratio = (Rp . Rf)/Standard
Deviation; Hence 0.04; Jensen = Rp-[Rf + B(RM .RF)] ; It measures the risk adjusted portfolio return. It is
also known as Jensen.s Alpha; Hence -4.25

300. ABC Ltd. is willing to prepay your Cumulative Fixed Deposit with them, without any penalty and
with all the accumulated interest (compounded half yearly). You had invested Rs. 4000 with them 3.5
years back. If they are giving you back Rs. 4985, what is the annualized rate of interest you have earned?

a) 6.40%
b) 3.2%.
c) 6.5%.
d) 7.2%.

Solution: (((((4985/4000)^(1/7))-1)*100)*2)

301.. Which of the following is a tort of negligence?

Investment Planning – 19Oct 07 54


a) Mr. Joy was playing golf. He swings a new golf club on the fairway and the head of the club
flies off, and hit another golfer who was standing 20 feet away.
b) Mr. Vishal takes medication that he knows makes him drowsy and then proceeds to
drive. He gets into an accident injuring the passengers in another car.
c) Mrs. Jaya locks Ms. Rani in a room to prevent him from leaving the building
d) Mrs. Priti experienced a sudden surge of chest pain while driving, which causes her to lose
control of her car and hit another car.

302.. Any possible occurrence which may have a negative financial implication, can be plotted on a graph
with X axis measuring the frequency (low-high) and Y axis measuring the financial impact (low-
high). You can view the classification in four quadrants.

Quadrant I - Low frequency, Low Impact


Quadrant II - Low frequency, High Impact
Quadrant III - High frequency, High Impact
Quadrant IV - High frequency, Low Impact

It would not be practical to purchase insurance for events falling in _________________.

a) Quadrant I & IV
b) Quadrant I, II & IV
c) Quadrant I, III & IV
d) Quadrant III

303. _____________funds pay a Dividend Distribution tax on dividends.

a) Equity
b) Index
c) Debt

304. The current annual dividend of ABC Corporation is Rs.2.00 per share. Five years ago the dividend
was Rs.1.36 per share. The firm expects dividends to grow in the future at the same compound annual
rate as they grew during the past five years. The required rate of return on the firm's common stock is
12%. The expected return on the market portfolio is 14%. What is the value of a share of common stock
of ABC Corporation using the constant dividend growth model? (Round to the nearest Rupee.)

a) Rs.11.
b) Rs.17.
c) Rs.25.
d) Rs.54
Solution:
PV = Rs.1.36, FV = (Rs.2.00), N= 5, i = 8.02
Value of common stock = d1/ RRR – g = 1.082 (2.00)/ 0.12 – 0.08 = 2.16/0.04 = Rs.54.00
RRR = Required Rate of Return, g = growth rate

305. Bond A has a 6% annual coupon and is due in 2 years. Its value in today's market is Rs. 900.
Bond B has a 10% annual coupon and is due in 4 years. It is priced to yield 12%.
Bond C is a 9% zero-coupon bond priced to yield 11% in 8 years.

The yield to maturity of Bond A is closest to:

a) 9.90%.
b) 10.40%.
c) 10.90%.
d) 11.90%.

Solution: Yield to maturity for bond A:


N = 2, PV = (Rs.900), PMTOA = Rs.60, FV = Rs.1,000, i = 11.90998

Investment Planning – 19Oct 07 55


306. Bond A has a 6% annual coupon and is due in 2 years. Its value in today's market is Rs.900.
Bond B has a 10% annual coupon and is due in 4 years. It is priced to yield 12%.
Bond C is a 9% zero-coupon bond priced to yield 11% in 8 years.
Assuming that the duration of Bond A is 1.94 years, which of the following statements about the
effect of a 1% decline in interest rates is true?

a) Bond C, having a longer duration than Bond A, would have a larger percent increase
b) The percent change in price of a bond is independent of the duration of a bond.
c) It is not possible to determine the percent change in price of Bond A versus Bond C
d) Bond A would have a greater percent change in price than Bond C because it has a shorter
duration.

307. A perfectly diversified portfolio will fully eliminate ______________ risk.

a) Systematic
b) Unsystematic

308. __________________measures the caliber of the fund manager.

a) Beta
b) Delta
c) Alpha

C:\WINDOWS\TEMP\quiz-apr-inv3.html

309. The new Senior Citizen.s Savings Bond Scheme offers ________ % Interest.

8.5
9
9.25
8.75

310. The maximum amount that can be invested in Public Provident Fund is Rs._________.

a) 70000
b) 60000
c) 80000
d) 90000

311. _____________ is regulated by the Reserve bank of India.

A. Bank Deposit Rates; B. Bank Lending Rates; C. Certificate of Deposit Rates

a) A
b) B
c) C
d) None

312. The recent Union Budget exempts ______ from Long Term Capital Gains tax subject to certain
conditions.

a) Equity Shares
b) Debt Mutual Funds
c) Property
d) Gold

313. The deduction from Gross Total Income available u/s 80E is Rs.___________.

a) 40000
b) 45000
c) 50000
Investment Planning – 19Oct 07 56
314. _________ is / are governed by SEBI.

a) Mutual Funds
b) Stock Brokers
c) Portfolio Managers
d) All of the above

315. A person can be qualified as an Associate Financial Planner after he/she passes modules
of the CFP Certification Course.

a) 6
b) 2
c) 3
d) 5

316. The first step of the financial planning process is ___________.

a) Evaluating the various Alternatives


b) Data gathering and goal setting
c) Establishing the Client Planner relationship
d) Plan Review

317. How many years will it take for a sum of Rs.10000 to double if the rate of return is 9% p.a.?

a) 9.5
b) 8.5
c) 10
d) 9
e) 8

318. If the post tax rate of return on an investment is 8% and the inflation rate is 5% the real rate of
return is___

a) 3.5%
b) 3.0%
c) 2.86%
d) -3.0%
e) 2.74

319. The key reference rate for inter-bank overnight borrowings in Mumbai is known as
a) MIBOR
b) MIBID
c) LIBOR
d) LIBID

320. This relationship between the trustees & the unit holders of a mutual fund shareholders is called a
__________ relationship

a) Contractual
b) Fiduciary
c) Moral obligatory
d) None of the above

321. Selling the wrong type of policies in order to earn higher commissions will render an insurance agent
to be held liable ______________

a) Under law of tort


b) for professional misconduct
c) for professional negligence
d) under the SEBI Act
Investment Planning – 19Oct 07 57
322. In portfolio management, a constant ratio plan is one that

a) Restores asset holdings at the end of a period to target weights.


b) Buys different kinds of securities in a fixed proportion.
c) Pays out dividends in a constant ratio to the portfolio‟s asset values.

323. As interest rates rise bond prices _____ and the longer their maturities the _______ the price
change.
a) Rise; lesser
b) Fall; lesser
c) Fall; greater

324. Which item below is not a method recommended for acquiring securities?

a) Dividend reinvestment plans


b) Rupee cost averaging
c) Averaging alpha and beta weights

325. KM, Inc. has a beta of 1.5. The risk-free rate is 5%, and the market risk premium is 8%. Thus,

a) KM has a required return of 6.5% and an expected return of 8%.


b) KM has a required return of 15.5%.
c) KM has a required return of 17.0%.

326. NASDAQ is
a) A section of the NYSE where technology stocks are traded.
b) The trading symbol for a racetrack company listed on the NYSE.
c) An electronic trading system used by securities firms.

327. An equity mutual fund has an NAV of Rs. 15 on January 1, 2006. You invest Rs. 15000 on that day.
You sell these units at an NAV of Rs. 25 on January 15, 2007. The entry load is 2.25% while the exit load
is 0.50%. You are in the 30% tax bracket. Your post-tax return on the same is _________ %

a. 66.66
b. 62.18
c. 46.66
d. 42.18

328. The thumb rule states that we should choose an investment with a _______ IRR.

a. Lower
b. Higher
c. Equal
d. Data insufficient

329. Seema and Arun are co-applicants of a mortgaged house. They are on the verge of a divorce. The
Housing Finance Company will ___________.

a) not interfere as long as the EMIs are being paid on time


b) repossess the house after divorce
c) insist on the house being transferred to one of them
d) mediate reconciliation between the couple.
e) Increase the interest rate in order to compensate for the increased risk

330. Domestic GOI bond holders (holding them up to Maturity) have to deal with ___________ risk.

a) Volatility
b) Default
c) Inflation
Investment Planning – 19Oct 07 58
d) Price
e) Currency

331. Refinancing is ______________.

a) Borrowing at lower cost in order to pay off higher cost debt


b) Repaying debt by selling off assets
c) Lending at a higher rate of interest
d) Securitizing your receivables
e) None of the above

332. ________________ Asset Allocation is not a text book Asset Allocation Model.

a) Tactical
b) Discretionary
c) Strategic
d) All of the above
e) None of the above

333. Jack and Jill approach you to be their Financial Planner their funds are limited and their needs are
many. Some of their needs are:

a) To start an investment plan for funding their child.s education;


b) To set up a Testamentary Trust for their child;
c) To set up a contingency fund amounting to 3 months of living expenses
d) To start saving for retirement;
e) To purchase life and health insurance.
Arrange these needs in the descending order of priority.

a) ceadb
b) debca
c) bdeac
d) beacd

334 For a nominal interest rate of 6% payable monthly, quarterly, and semi-annually, the effective
rates respectively ould be _______________.

a) 6.04, 6.02, 6.01


b) 6.16, 6.13, 6.09
c) 6.10, 6.07, 6.03
d) 6.11, 6.08, 6.06

335. A 10 year 9 % Bond (Face Value of Rs.100, interest payable annually) maturing 3 years from today
is available at a YTM of 5.8%. Therefore the current price is ___________.

a) 152.50
b) 154.10
c) 153.32
d) 152.00

336. A one-time principal repayment for a bond is known as a ____________ repayment.

a) Laddered
b) Step-Up
c) Step-down
d) Bullet

337. Arrange the following financial planning functions into the logical order in which a professional
financial planner performs these functions.

1. Interview clients, identify preliminary goals.


2. Monitor financial plans.
3. Prepare financial plan.
Investment Planning – 19Oct 07 59
4. Implement financial strategies, plans, and products.
5. Collect, analyze, and evaluate client data.

a) 1, 3, 5, 4, 2.
b) 5, 1, 3, 2, 4.
c) 1, 5, 4, 3, 2.
d) 1, 5, 3, 4, 2.
e) 1, 4, 5, 3, 2.

338 A model that describes the relationship between risk and expected return and that is used in the
pricing of risky securities is better known as _________

a) Efficient Market hypothesis


b) CAPM
c) Security Market Line
d) Beta Model

Four Marks Questions


For question no. 339 and 340.The stock of Maxi Limited performs relatively well to other stocks during
recessionary periods. The stock of Taxi Limited, on the other hand, does well during growth periods. Both
the stocks are currently selling for Rs.100 per share. You assess the rupee return (dividend plus price) of
these stocks for the next year as follows:

Economic Condition
High Growth Low Growth Stagnation Recession
Probability 0.3 0.4 0.2 0.1
Return on 100 110 120 140
Maxi‟s stock
Return on 150 130 90 60
Taxi‟s stock

Calculate the standard deviation of investing:


339. Rs.1, 000 in the equity stock of Maxi Limited

a) 120.5
b) 124.6
c) 116.6
d) 123.4

340. Rs.1, 000 in the equity stock of Taxi Limited

a) 301.4
b) 322.6
c) 291.4
d) 296.8

Investment Planning – 19Oct 07 60


341. Rs.500 each in the equity stock of Maxi Limited and Taxi Limited.

a) 90.6
b) 104.6
c) 78.4
d) 89.6

342. You purchase One Contract of the September Call option for Dr. Reddy‟s Labs at Rs 1400 strike
price for a premium of Rs. 100 . The price on the date of contract expiry is Rs. 1600. The market lot is
200 shares. Your profit on the above transaction is _______ %. Ignore transaction costs.

a) 50
b) 25
c) 15
d) 10

343. Sanjeev invests Rs.5000 in a Bank Deposit today @ 8% p.a compounded monthly.. He
hopes that this investment will enable him to fund his college education (estimated to cost Rs.9000)
which commences after 4years. What will be the value of this investment in four years?

a) 6802
b) 6870
c) 6878
d) 6925

344. Sudha invests Rs.5000 per year (at the beginning of each year) for 5 years @ 5% p.a. in a bank
deposit. She then withdraws the accumulated sum over a period of 3 equal annual installments. What is
the value of the deposit at the end of 5 years and the quantum of withdrawal each year?

a) 28505, 9954
b) 29010, 10656
c) 29568, 11054
d) 28804, 10042

345. Amar wants to purchase a Car 5 years from now. His investments are currently worth
Rs.50,000/- and he intends to contribute Rs.10,000/- at the beginning of every six months period to fund
his purchase. Assuming that the annual investment rate of return is 8% compounded semi
annually, what will be the value of the investment in five years time?

a) 1,98,875
b) 1,95,555
c) 1,97,240

346. Neeta wants to accumulate Rs.1, 50,000 in three years time for a one month trip to the USA.
Assuming she can get an 8% annual return on her investments, compounded quarterly, how much must
she invest today in order to achieve her goal?

a) 117591
b) 119487
c) 118274

347. John has estimated that the following will be his outgoings over the next few years:
End of Year Cash Outflow

a) 1st Rs.10000
b) 2nd Rs.15000
c) 3rd Rs.12000
d) 4th Rs.13500
e) 5th Rs.11000

348. If John wants to cater to these cash outflows, how much should he have today, assuming an annual
rate of return of 5%?

Investment Planning – 19Oct 07 61


a) 54126
b) 53220
c) 52483
d) 50483

349. Mr. John has purchased 100 convertible debentures of Essar Oil on 1/1/94 at Rs.500 each. 40% of
the value of the debentures is convertible into one share of Rs.50 each after seven years. Mr. John
exercised his option on 1/4/2001 and received 100 shares. Compute the cost of acquisition of these
shares.

a) 200
b) 205
c) 195
d) 185

350. Revenue over the past four years are given below:
st
1 year 100
nd
2 year 120
rd
3 year 150
th
4 year 200

Calculate the CAGR:

a) 20%
b) 18.9%
c) 25%
d) 25.9%

Source: Quiz- Aug – Inv 3

351. The maxim “buy term and invest the balance” may not be a feasible proposition for many investors
for the following reasons EXCEPT for:

A. it may be difficult to achieve sufficient diversification in the invested assets


B. it may be difficult to achieve a suitable investment portfolio with the desired risk reward relationship
C. it may be possible to consistently outperform the investment returns earned by an established
life office
D. that the insurer offers capital guarantees on cash values

352. An individual may prefer investment-linked assurance to conventional assurance policies for the
following reasons EXCEPT

A. he has some direction over the investment of his premiums


B. he is informed of the expenses charged for the services provided
C. he is attracted by the guaranteed surrender values offered under these contracts
D. he prefers the switching facilities available under such contracts

353. Which of the following does not constitute a valid charge on the premiums
paid for an investment-linked life assurance policy?

A. bid-offer spread
B. front-end charges
C. recurrent fund related charges
D. surrender penalty

354. You are an independent financial planner and are faced with the problem of suggesting a suitable
insurer for a client who is in very good health. Assuming that a temporary assurance policy is needed for
this client, you would recommend an insurer

Investment Planning – 19Oct 07 62


A. with a stringent underwriting policy
B. with a lax underwriting policy
C. with very few causes of death excluded
D. with extensive causes of death excluded

355. Identify the class of assurance for which an insurer, in a costing exercise, must provide for future
anticipated improvements in mortality

A. investment-linked endowment assurance


B. conventional endowment assurance
C. group life assurance
D. life annuities

356. A type of risk with high frequency but low severity is probably best handled by:
a) Self-insurance
b) Insurance
c) Avoidance
d) Transfer

357. 'Consideration' under the law is a return promise to:


(1) Do certain things
(2) Abstain from doing certain things
(3) Forbear some acts
(4) Accept an offer

a) (1), (2), and (3) only


b) (1), (2), and (4) only
c) (1), (3), and (4) only
d) (2), (3), and (4) only

358. Frank has an indemnity policy in respect of his house. The house was valued when he first took out
the policy three years ago at Rs.80,000 and insured for that amount. The policy has been
renewed each year since, without alteration. If the house is destroyed by fire and the cost of rebuilding it
would be Rs.110,000, how much is Frank likely to recover?

a) Rs.58,182
b) Rs.80,000
c) Rs.88,000
d) Rs.110,000

359. A client explains that she only wants a life insurance policy that will cover her family against
financial risk over the next five years, while she still has dependent children and a large
mortgage. It is unlikely her income will increase over this period. What type of insurance is she
looking for?

a) Pure endowment assurance


b) Term insurance with a level premium
c) Single premium life insurance
d) Whole of life assurance to be made paid up after five years

360. The 'principle of subrogation' means:

a) An insurer may bring an action against a third party in the name of the insured after satisfying a
claim
b) An insured may not take legal action in respect of a matter which is the subject of a claim
c) An insurer undertakes to take legal action on behalf of its insured and to pay to the insured
the net proceeds of such legal action
d) An insurer which has satisfied a claim may recover from the insurer of a third party who
admits liability

Investment Planning – 19Oct 07 63


361. Which of the following is true regarding the ownership of life insurance?

a) A policy can only be issued to the insured


b) Generally, assigning a policy requires proof that the insured is still „insurable‟
c) Only a person with an insurable interest can be named as a beneficiary
d) The owner can assign the policy to whomever he or she chooses, even if the assignee has
no insurable interest

362. Prakash has a premium due on a motor insurance policy. He calls the agent and requests for an
extension. The agent assures him that the insurer usually gives a grace period of one week for
such payment. Prakash has an accident. The insurer cannot deny liability for the loss on the
grounds that the premium was not paid because of the doctrine of:

a) Waiver
b) Estoppel
c) Agency
d) Utmost good faith

363. A reversionary bonus is:

a) Any profit distributed to holders of life policies who cash in those policies
b) Generally profit that will be distributed on the same terms as the sum assured
c) A distribution of income to shareholders
d) Profit distributed to holders of unit-linked policies

364. Professional indemnity insurance protects a financial advisor who has been negligent in giving
investment advice:

a) Only if the advisor is liable under statute


b) Only if a contract exists between the advisor and the investor
c) Only if the investor relies on the advice
d) Only if the advisor has not included a disclaimer of liability in the contract with the investor

365. Deepika purchased a dining room set for Rs.2,800 and insured it on an actual cash value (ACV)
basis. At the time the dining room set was destroyed by a covered peril, the set was 40 percent
depreciated. A replacement set will cost Rs.3,000. Assuming no deductible, how much will Deepika
collect from her insurer?

a) Rs.1,800
b) Rs.3,000
c) Rs.1,880
d) Rs.2,800

366. Property insurance contracts have all of the following distinct legal characteristics EXCEPT

a) They are personal contracts.


b) They are contracts of adhesion.
c) They are bilateral contracts.
d) They are aleatory contracts.

367. In which section of an insurance contract will you find provisions that qualify or place limitations on
the insurer‟s promise to perform?

a) Exclusions
b) Definitions
c) Conditions
d) Insuring agreement

368. Which of the following statements is (are) true about the insuring agreement in an insurance policy?

i. The insuring agreement provides a description of the property or activity to be insured.


ii. The insuring agreement can be written on an “all risks” basis or on a “named-perils” basis.
Investment Planning – 19Oct 07 64
a) Both I and II
b) Neither I nor II
c) II only
d) I only

369. Saif and his wife Amrita are legally separated. The couple owns a vacation cabin. Saif purchased a
Rs.25,000 property insurance policy on the cabin. Unaware that Saif had purchased this coverage, Amrita
purchased a Rs.50,000 property insurance policy on the cabin. While both policies were in force, a
Rs.12,000 covered loss occurred. The insurers agreed to settle the claim on a pro rata basis. What is
each insurer‟s liability?

a) Saif‟s insurer pays Rs.3,000 and Amrita‟s insurer pays Rs.9,000.


b) Saif’s insurer pays Rs.4,000 and Amrita’s insurer pays Rs.8,000.
c) Saif‟s insurer pays nothing and Amrita‟s insurer pays Rs.12,000.
d) Saif‟s insurer pays Rs.12,000 and Amrita‟s insurer pays nothing.

370. Under the needs approach of determining the amount of life insurance to purchase, one
consideration is providing income to the surviving spouse and children during the one- or two-year period
following the breadwinner‟s death. This period is called the
a) Accumulation period.
b) Dependency period.
c) Readjustment period.
d) Blackout period.

371. Sheila would like to purchase cash value life insurance policy. She is concerned, however, that if
she becomes disabled she will be unable to pay the premiums as they come due. What provision can
Sheila add to her policy to address this concern?

a) Reinstatement provision
b) Waiver-of-premium rider
c) Accelerated death benefits rider
d) Guaranteed purchase option

372. Which of the following statements is (are) true with respect to life insurance policy loans?
i. Interest must be paid on life insurance policy loans.
ii. If a policy loan has not been repaid when the insured dies, the outstanding loan balance I
s deducted from the proceeds paid to the beneficiary.

a) II only
b) I only
c) Neither I nor II
d) Both I and II

373. Which of the following statements is (are) true with respect to the traditional net cost method of
determining the cost per thousand of cash value life insurance?
i. The traditional net cost per thousand per year can be a negative number.
ii. The traditional net cost method ignores interest that could have been earned on the premiums by
investing them elsewhere.

a) I only
b) II only
c) Both I and II
d) Neither I nor II

374. Bob and Jasmine is a married couple who are both 67 years old. Bob and Jasmine purchased an
annuity covering both of their lives. The settlement option will provide payments until Bob and Jasmine
are both deceased. The settlement option Bob and Jasmine selected is a (n)

a) Installment refund option.


b) Life income with guaranteed payments option.
c) Cash option.
d) Joint-and-survivor annuity option
Investment Planning – 19Oct 07 65
375. Shelly purchased a major medical policy. The policy has a calendar-year deductible of Rs.500 and
80-20 coinsurance. Shelly was hospitalized with a covered illness on January 23rd. This hospitalization
was his first claim under the major medical policy for the calendar year. His covered medical expenses
were Rs.20,500. How much of this amount will the insurer pay, and how much will Shelly be required to
pay?

a) The insurer will pay Rs.16,900 and Shelly will pay Rs.3,600.
b) The insurer will pay Rs.16,000 and Shelly will pay Rs.4,500.
c) The insurer will pay Rs.15,900 and Shelly will pay Rs.4,600.
d) The insurer will pay Rs.20,000 and Shelly will pay Rs.500.

376. What lump sum contribution made now into a deferred annuity scheme will provide to 20-year old
males annual payments commencing at age 55 of Rs. 1,200 and escalating thereafter at an annual rate
of 2% per annum for the next 20 years, if the funds under such a scheme earn 6% per annum rate of
interest throughout?

A. Rs.2,221
B. Rs.1,898
C. Rs.2,137
D. Rs.2,002

377. Moral hazard is likely in insuring each of the following risks EXCEPT:

A. A company that had made many claims under its fire insurance policy
B. An individual purchasing life annuity at his retirement.
C. An individual purchasing life insurance on his own life
D. An individual purchasing life insurance on his wife‟s life

378. An insurance contract is voidable if it can be established that the policyholder

A. did not state his correct date of birth


B. did not provide precise information about his parents
C. died within the contestable period
D. did not furnish accurate information about his current state of health

379. The following are important factors in the determination of premiums for disability income policies
EXCEPT:

A. The elimination period


B. The definition of disability
C. Preexisting conditions
D. The insured‟s occupation

380. What lump sum contribution made now into a deferred annuity scheme will provide to 20-year old
males annual payments of Rs.1,200 commencing at age 55 for the next 20 years, if the funds under such
a scheme earn 6% per annum rate of interest throughout?

A. Rs.1790
B. Rs.1898
C. Rs.2006
D. Rs.3122

381. For an initial lump sum consideration of Rs.5,295 paid at age 20, a deferred life annuity scheme
offers the following benefits:
On attaining age 55, an annuity certain for 10 years and a life annuity thereafter, of Rs. 1,200 per annum.
Assuming the annuitant dies at his 75th birthday, calculate the internal rate of return implicit in the
promises made.

Investment Planning – 19Oct 07 66


A. 3.1 %
B. 3.3 %
C. 3.5 %
D. 3.7 %

382. One of the key criteria for the approval of applications made IRDA for the conduct of insurance
business in India, is that the applicant must have a large capital base. This large capital base is
necessary for the following reasons EXCEPT:

A. To absorb the losses that may result from charging inadequate premiums.
B. To absorb the catastrophic losses that may arise in some years.
C. Insurance business is capital intensive.
D. A large capital base is a pre-requisite for achieving economies of scale.

383. The following is an abridged version of the balance sheet of a large life insurance company:

Balance Sheet of ABC as at 31Dec, 2006 (Rs.‟000,000)


Liabilities
Shareholders‟ Funds 800
Statutory Liabilities 3,400
Current Liabilities 50
Total 4,250
Backed By
Indian Government Securities 700
Corporate Bonds 200
Ordinary Shares 2,050
Properties 800
Policy Loans 500
Total 4,250

A measure of the company‟s financial strength is:

A. Rs. 4,250,000,000
B. Rs. 3,450,000,000
C. Rs.3,400,000,000
D. Rs. 800,000,000

384. The proceeds of Rs.150,000 from a life insurance policy is to be disbursed under a settlement option
using an annuity due with term certain of 15 years. Assuming that a 4% per annum interest is appropriate,
the level annual payments arising under this arrangement is:

A. Rs.13,491
B. Rs.12,972
C. Rs.7,203
D. Rs.7,491

385. The waiting period under a disability benefit policy refers to

a. The period of time that must elapse before the policy benefit will commence
b. The period of time that must elapse before settlement of the claim after submission.
c. The period of time that must elapse before the issue of the policy document.
d. The period of time that must elapse before the proposal can be considered.

386. Seema and Arun are co-applicants of a mortgaged house. They are on the verge of a divorce. The
Housing Finance Company will ___________.

a) not interfere as long as the EMIs are being paid on time


b) repossess the house after divorce
c) insist on the house being transferred to one of them
d) mediate reconciliation between the couple.
Investment Planning – 19Oct 07 67
e) iIncrease the interest rate in order to compensate for the increased risk

387. In the event of a claim, an indemnity contract pays to the insured an amount equal to

A. the sum insured


B. the extent of loss subject to a maximum equal to the sum insured
C. the extent of loss without any limits
D. a previously agreed amount

388. For a disability income policy, which of the following factors are important in its premium
determination?

I. Replacement ratio II Definition of disability

A I only
B II only
C I and II
D Neither I nor II

389. In controlled Third party captive business, retailers are underwriting


a. Credit life, health and disability; extended warranty
b. Subcontractor coverages
c. Joint ventures
d. Credit life, health and disability; mortgage insurance, title insurance , auto and property.

Use the information. below to provide answers to Questions 390 to 392


A client of yours, who is now aged 35 years, states that his retirement objective is to provide for himself at
retirement at age 55, annual incomes of 2/3 of his last drawn salary. He further states that he expects this
income for 20 years after retirement. His current annual salary is Rs. 60,000 and this escalates at 6% per
annum throughout his career, with the annual escalations occurring on his birthdays.
Assuming an 8% per annum interest rate throughout,

390. Calculate his last drawn annual salary

A. Rs.128,400
B. Rs.181,536
C. Rs.192,428
D. Rs.132,000

391. Calculate the present value of the retirement income

A. RS.225,800
B. RS.250,900
C. RS.275,327
D. RS.305,780

392. Calculate the level annual amount that he must deposit until age 54, assuming
that the first is due now, to fund for the stated retirement income.

A. RS.22,800
B. RS.25,965
C. RS.28,890
D. RS.31,234

393. Participating ordinary life policies with a sum assured of Rs. 10,000 are issued by two life assurance
companies, Office 1 and Office 2, to lives aged 35 with the following cost data:

Data Office 1 Office 2


Rs. Rs.
Annual premiums 230 290
Accumulated value of 1613 1700

Investment Planning – 19Oct 07 68


Total bonuses for 20 years at
6% pa
Cash value at end of 20 years 3620 4000

394. Based on the above information and assuming a 6% per annum interest rate, the annual surrender
cost index for each Rs.1,000 sum assured at the end of 20 years

A. For Office 1 is greater than Office 2


B. For Office 2 is greater than Office 1
C. For Office 1 is equal to Office 2
D. Is not determinable

395. Calculate surrender value:

Date of commencement – 5.10.1990


Date of last premium – 5.4.2000
Date of birth – 1.5.1950
Mode of installment premium – half yearly
Plan – Endowment with profit – 50 years
Sum assured – Rs.1, 20,000
Bonus accrued from 3/91 to 3/2000 is Rs.700 per 1, 000 sum assured
Accidental benefit – Rs.2 per thousand sum assured
S.V. factor – 25%

a. 6000
b. 27,000
c. 6,240
d. 27,240

Solution: Paid-up value- No. of premiums paid X sum assured /No. of premium payable + bonus if any.
Surrender value

Paid-up value X S.V. factor/100


Paid-up value: 20 X 1, 20,000/100
= Rs.24, 000 + 84, 000 (bonus for 10 years Rs.700 per 1, 000 S.A)
= Rs.1, 08,000
Surrender Value: 1,08,000 X 25/100
= Rs.27, 000
Therefore surrender value is Rs.27, 000.

396. Calculate the premium:

Tabular premium: Rs.33.10 per sum assured.


: Rs.2 less for yearly mode.
: Re.1 less for half yearly mode.
: Rs.3 less for sum assured Rs.1, 00,000 and above.

Double accident is allowed up to a maximum of Rs.10lakhs sum assured on payment of Re.1 per 1000
sum assured. Calculate yearly premium for Rs.15 lakhs sum assured with occupation extra of Rs.4 per
thousand sums assured plus double accident benefit is allowed.

a. 49,150
b. 48,150
c. 43,150
d. 52,150

Solution:

Investment Planning – 19Oct 07 69


Tabular premium Rs.33.10
Rebate allowed are yearly mode (Rs.5.00)
Rs.2 large sum assured Rs.3 Balance Rs.28.10
Balance X sum assured / 1000 Rs.28.10 X 15, 00,000/1000 =
Rs.42, 150
Plus occupation extra 4 X 1500 Rs.6000
Plus double accident benefit 1 per 1000 – since
maximum double accident benefit allowed in up
to Rs.10, 00,000 sum assured Rs.1, 000
Total Rs.49, 150
Yearly installment premium is Rs.49, 150.

Based on the above information and assuming a 6% per annum interest rate, the annual surrender cost
index for each Rs.1,000 sum assured at the end of 20 years

A. For Office 1 is greater than Office 2


B. For Office 2 is greater than Office 1
C. For Office 1 is equal to Office 2
D. Is not determinable

397. A group of 60000 persons each aged 45 years wishes to apply for term insurance for a one year
period for a sum of Rs. 2,80,000. If mortality tables show that out of 80,00,000 people 40,000 die within a
year, find the premium to be paid by each of the 60000 applicants.
a. 1500
b. 1250
c. 1400
d. 2750

398. Calculate HLV to recommend adequate insurance cover; Mr. Satish, Age= 30 yrs Retirement age 65
years. He is senior Manager. His monthly salary is 10,00,000. he pays professional tax of rs.5,000 and
income tax subject to allowable deductions i.e. tax paid Rs. 1,95,000 Reasonable self maintenance
expenditure estimated Rs.1,00,000 p.a.; life insurance premium for self Rs. 20,000 with total sum assured
Rs. 20,00,000. For wife and child he pays insurance premium of Rs. 13000 and Rs. 7000 respectively
rate of interest assumed for capitalization of future income is at 10%. Adequate additional insurance
recommended is
a) Rs.65 lakh
b) Rs.68 lakh
c) Rs.88 lakh
d) Rs.86 lakh

399. The following are statements concerning contracts of insurance. Identify the
statement/s that is/are correct.

I. Whenever the wording in an endorsement or rider is in conflict with the terms of the policy to which it is
attached, the endorsement or rider takes precedence.
II. One reason for exclusions in insurance policies is that the risks are covered by other insurance.

A. I only
B. II only
C. I and II
D. Neither I nor II

400. Mohan invested Rs. 420000 for 7 years @ 7% where it was compounded annually for the
first 5years and quarterly for the last 2 years. What did he receive on maturity?

a) 676774
b) 776774

Investment Planning – 19Oct 07 70


c) 931095
d) 609870

Quiz Inv Class Test 1

Type 1 one mark

401. The correlation co-efficient of two perfectly correlated assets is _____

a. +1
b. -1
c. +0.5
d. -0.5

402. A zero coupon bond will have zero_______ risk

a) Interest Rate risk


b) Inflation risk
c) Reinvestment risk
d) Default risk

403. Shyam has a one year old daughter. He wants to save for her higher education which shall begin at
age 21. Which investment vehicle has the best potential for him to attain his goals?

a) Derivative Instruments
b) GOI Bonds
c) Bank Fixed Deposits
d) Equity Mutual Funds

404. ______________ is a form of Secured Loan

a) All of the above


a. Loans against shares
b) Credit Card Debt
c) Holiday Loans

405. Beta is a term commonly used in the __________

a. Art market
b. Real Estate market
c. Fixed Income Market
d. Stockmarket

406. Bank Fixed Deposits eligible for 80 C deduction must have a minimum tenure of _______ years

a) 3
b) 4
c) 5
d) 6

407. ____________ bonds are eligible investments u/s 54 EC

a) NHAI
b) NABARD
c) REC
d) Both A and C

408.__________ is an asset which is exempt from Long Capital Gains subject to certain conditions

Investment Planning – 19Oct 07 71


a) Equity Share
b) Real Estate
c) Art
d) Commodities

409 _____________ are generally the preserve of the Wealthy class as compared to the Middle class

a) Fixed Deposits
b) Mutual Funds
c) Equity Shares
d) Collectibles

410. The Mumbai Stock Exchange is a ____________

a) Corporate entity
b) Partnership Firm
c) Association of Persons
d) None of the above

411 ______________ funds have the lowest expense ratio

a) Equity
b) Gilt
c) Index
d) Liquid

412 Your residence is treated as a ____________ asset

a) Personal Use
b) Investment
c) Near-Cash
d) None of the above

413. The yield curve is usually _________________

a) Upward sloping
b) Downward sloping
c) Flat
d) Humped

414. Value investing involves purchasing stocks with __________

a) Low P/E Ratios


b) Low Book Values
c) Low Dividend Yields
d) Low Margin of safety

415. Narrow money does not comprise of _____________


a) Currency with the public
b) Demand deposits with the banking system
c) Other deposits with the RBI
d) Time Deposits with banks

416. The rate at which commercial banks and other lending facilities can borrow short-term funds from the
central bank is called the

a) Discount rate
b) Repurchase Rate
c) Reverse Repurchase Rate
d) Prime Lending Rate

417. The effective interest rate earned per rupee _______ as the periods of compounding increase.

Investment Planning – 19Oct 07 72


a) Increases
b) Decreases
c) Remains same
d) Decreases for some time and then increases
e) Data insufficient

418. Sharpe ratio determines return per unit of _______________

a) Diversifiable Risk
b) Non-diversifiable risk
c) Both of the above
d) None of the above

419 Treynor ratio uses _______ as the divisor.

a) Beta
b) Standard Deviation
c) Variance
d) Either b. or c.

420. Jensen‟s ratio helps in determining __________

a) Alpha
b) Beta
c) Gamma
d) Vega

421. SEBI is the Regulator for _____________ companies

a) Listed
b) Unlisted
c) Both A and B
d) Foreign

422 An important characteristic of gilt securities are that they ___________.

a) are issued by the Central Government.


b) all have terms to maturity that are 270 days are less.
c) all tend to have large amounts of purchasing power risk.
d) are issued by corporates

423. One standard deviation means that an observation will fall between the upper and lower bound
______ % of the time.

a) 50.25%
b) 68.30%
c) 75.5%
d) 95%

423. The statistical measure of covariance is known as _____________

a) Standard Deviation
b) Correlation coefficient.
c) Variance
d) Level of confidence

424. ________ is a measure of systematic risk

a) Correlation coefficient
b) Beta
c) Alpha

Investment Planning – 19Oct 07 73


d) Sharpe Ratio

425. An investment in equity shares of Russian companies is most probably, not subject to ________

a) Currency Risk
b) Political Risk
c) Default Risk
d) Liquidity Risk

426. Profits earned through trading exchange traded options are treated as ________ gains.

a) Short Term Capital


b) Speculative
c) Long Term Capital
d) None of the above

427. The features of collectibles include ________

a) The fact that they can be easily valued.


b) The fact that they are traded on a Collectible Exchange
c) The fact that they have the potential for great capital appreciation.
d) The fact that common people can invest in them.

428. __________ is a widely used metric for measuring volatility of an investment.

a) Mean
b) Median
c) Standard Deviation
d) Variance

429. The IRR method assumes that all cash flows are reinvested at __________

a) The cost of capital


b) The market rate of return
c) The prevailing PLR
d) The rate decreed by the RBI

430. A rating of AAA (ind) by Fitch indicates that the particular security is free from _________ risk

a) Interest rate risk


b) Default risk
c) Inflation risk
d) Reinvestment risk

431. The YTM of a Bond is also known as its ______________

a. Internal Rate of Return


b) Holding period Return
c) Current Yield
d) None of the above

432. Equity Linked Savings Schemes eligible for Sec. 80 C benefits have a lock-in period of ____ years.
a) 2
b) 3
c) 4
d) 5

433. The most appropriate investment for a highly risk averse investor aged 57 is __________

Investment Planning – 19Oct 07 74


a) An index fund based on the Nifty.
b) A diversified equity fund
c) A Maharashtra State Government Bond
d) A SBI Fixed Deposit

434 The thumb rule states that we should choose an investment with a _______ IRR.

a) Lower
b) Higher
c) Equal
d) Data insufficient

435. Yield to Call is usually calculated for a ________

a) Debt Instrument with an option


b) Option free instrument
c) Derivative instrument
d) Data insufficient

436. The relationship between price and interest rate is _____________

a) Linear
b) Convex
c) Random
d) Concave

437. The following is not correct about a Gilt Mutual Fund.

a) They primarily invest in securities issued by the Central Government.


b) They primarily invest in longer dated instruments.
c) They are not subject to Capital Gains Tax if held for a period of over one year.
d) They are more volatile as compared to liquid funds.

Type 2 – Two Marks

438. You are in the process of designing a portfolio of securities for your clients. While emphasizing the
importance of diversification in the portfolio, you will explain that this should aim to ______________.

a) Completely eliminate both systematic and unsystematic risks.


b) Completely eliminate systematic risks.
c) Reduce unsystematic risks to the best possible extent
d) Increase return at all costs.

439. When we take into account only negative deviations from the mean, we term it as ___________

a) Standard Deviation
b) Variance
c) Semi-variance
d) Mode

440. How many years (approx.) will it take for a sum of Rs.10000 to quadruple if the rate of return is 9%
p.a.

a) 18
b) 16
c) 14
d) 12

441. If the post tax rate of return on an investment is 8% and the inflation rate is 5% the real rate of return
is

Investment Planning – 19Oct 07 75


a) 3.5%
b) 3.0%
c) 2.86%
d) -3.0%

442. Geeta invests Rs. 2000 at the beginning of each month for 48 months. Her rate of return is 8% p.a.
The investment‟s value at the end of the said period will amount to ___________

a) 1,10,000
b) 1,12,700
c) 1,15,000
d) 1,11,500

443. Suppose you invest in 4 securities. Company A has an expected return of 20%. Company B 10%.
Company C 12% and Company D 9%. You have invested Rs. 40,000. What more information is needed
to find out the return on the portfolio?

a) Beta of these shares


b) Proportion of investment
c) Market Value of the investments
d) None of the above

444. One standard deviation means that an observation will fall between the upper and lower bound
______ % of the time.

a) 50.25%
b) 68.30%
c) 75.5%
d) 95%

445.If Standard deviation is 8, the variance is equal to ________

a) 56
b) 64
c) 72
d) 80

446. _________ distribution, indicates that an a particular investment has a probability of one potentially
large loss as against several potentially small gains.

a) Lognormal
b) Positively skewed
c) Negatively skewed
d) Normal

447. The P/E Ratio of the Sensex is 15. The earnings yield is ______

a) 5.50
b) 6.25
c) 6.67
d) 7.05

448. A bank deposit of Rs. 25000 will earn an interest of Rs._____ at the end of one year, if it earns
10% p.a. compounded every month.

a) 2599
b) 2617
c) 2745
d) 2799
Investment Planning – 19Oct 07 76
449. A bondholder buys a bond for Rs.105 and earns Rs.10 p.a. as interest. His current yield is ______ %

a) 9.85
b) 9.69
c) 9.52
d) 9.39

Solution = (10/105*100 = 9.52%)

450. A bondholder buys a bond maturing in one year for Rs. 90 and earns Rs. 5 per annum as interest.
His Holding period yield is _______ %

a) 15.85
b) 16.67
c) 17.25
d) 18.19

Solution = (100-90) + 5 / 90*100 = 16.67%

451. John has an investment with an annual income of Rs.100 and current value of Rs.6,000. If the value
of the market is expected to rise to Rs.7200 by end of 3 years, the approximate yield on the investment is
_______ %
a) 7.25
b) 7.58
c) 7.85
d) 8.02
Solution
future price current price
annual income
number of years
Formula : Approximate yield =
future price current price
2
452. ____Losses suffered in stock market related transactions can be carried forward for upto ___ years.

a) Short Term Capital, 8


b) Speculative, 8
c) Long Term Capital, 8
d) Both a. and b.

453 Ram is an income seeking who is low on the risk curve. His Financial Planner suggests that he
invest in a Company Fixed Deposit rated FB by CRISIL, which is offering a yield of 300 basis points over
the FD rated FAAA. His advice is ________

a) Appropriate because Ram is able to earn high interest.


b) Appropriate because Ram is low on the risk curve.
c) Inappropriate because Ram is low on the risk curve
d) Inappropriate because Ram can earn even higher income by investing in a FD rated FC.

454. Senior Citizens Bonds offer _____ % interest and are eligible for _______ tax deduction

a) 8.5, 80C
b) 9.0, Nil
c) 9.5, 80C
d) 8.75, Nil

455. A company offers a rights issue of one for three for Rs.5 each. The present share price is Rs.13. If
the share price does not change during the time of trading, what is the price after the rights are taken up?

a) 11.00
b) 12.50
Investment Planning – 19Oct 07 77
c) 10.00
d) 10.50

Solution: 13*3 = 39 + 1*5 = 44. 44/4 = Rs. 11.00

456. A bondholder buys a bond maturing in two years for Rs. 120 and earns Rs. 15 per annum as
interest. His YTM is _______ %

a) 4.00
b) 4.55
c) 4.75
d) 4.95
Solution:
Bond' s par value - Current bond price
Annual interest
Number of years to maturity
Yield to maturity (YTM)
Bond' s par value Current bond price
2

Therefore YTM = {15 + (-20/2)} / {(100+120)/2} = 4.55 %

457. The net asset value (NAV) of a closed end fund has risen from Rs.52 to Rs.62. The fund was last
quoted at Rs40, prior to the announcement of the increase in the NAV. If the fund were to trade at the
same discount to NAV, estimate the new price of the fund?

a) 48
b) 52
c) 54
d) 57

Solution: The price is calculated as follows: (62 – 52)/52 = 19.2% 40 x 1.192 = 48

Investment Planning – 19Oct 07 78

You might also like